Современная наука, и физика в частности, имеют в своей основе одну, как я полагаю, ложную аксиому. И если её поправить, то всё научное мировоззрение значительно изменится и многое станет гораздо более ясным. Это аксиома о том, что человеческие способности познания Мира безграничны и рано или поздно, при более совершенных приборах и способах исследования, мы познаем всё и до конца. А наше знание о Мире абсолютно объективно.

Вместо неё, на мой взгляд, должна быть следующая аксиома: человечество случайно возникло в более-менее стабильной (временно) части бесконечного Мира. Познание Мира человечеством ограниченно и условно. Люди никогда не познают Мир до конца и всегда будут использовать условные, понятные лишь им самим, символы для описания Мира, с целью продержаться в нём ещё какое-то время, пока внешние глобальные условия позволяют это делать. Ограниченность и условность в познании Мира относится не только к человечеству, но и к любым другим возникающим в Мире субъектам.

Когда-то люди считали Землю центром, вокруг которого вращаются и Солнце и звёзды. Коперник разрушил такое представление людей об устройстве Мира. Пришло время разрушить ещё одно неверное представление, о том, что человек это существо, способное генерировать абсолютно объективное знание о Мире с помощью своего сознания.

Я полагаю, что «человечество» лишь один из многочисленных видов населяющих Мир существ, восприятие и познание Мира которыми всегда субъективно и действительно лишь для узкого круга примерно подобных им созданий. Восприятие нами Мира ограничено нашей человеческой сущностью, которая сама по себе условна и до сих пор нам не совсем понятна.

В связи с этим мы должны считать все наши достижения в познании Мира условными, ограниченными и так будет всегда. Они конечно способны как-то улучшать нашу жизнь, и мы вполне ими можем гордиться, но должны понимать, что нам никогда не вырваться из определённых наших сущностных, временных и локальных ограничений. Абсолютно объективное знание о Мире невозможно, это просто миф.

Исходя из всего этого, я предлагаю по новому взглянуть на некоторые положения современной науки и физики в частности.

Материя, поля, тёмная материя, тёмная энергия и прочие «условности»

Бастиан Ван Фраассен, голландско-американский философ (ХХ век): «Внутри науки проводится различение между наблюдаемым и ненаблюдаемым. Это антропоцентрическое различение».     

Освальд Шпенглер немецкий философ (XX век): «Всякий  научный  опыт, каким бы он ни был, является, ко всему прочему, еще и свидетельством способов  символического представления». 

Полагаю, что каждый субъект воспринимает Мир своим особенным, уникальным способом. Но восприятие различными субъектами Мира может быть примерно одинаковым (как указывал ещё Иммануил Кант). И вот эту общую в их восприятии часть Мира данные субъекты начинают считать «относительно объективной» и присваивать различным её условным формам условные символы в общении между собой.

Наиболее общедоступную и хорошо воспринимаемую нами часть Мира, мы, субъекты-люди обозначаем как «материальный мир». Но, по-видимому, большая часть Мира от нас, субъектов-людей, скрыта и нам практически недоступна. Тем не менее, она оказывает на нас определённое влияние. Мы обозначаем эти слабо или почти невоспринимаемые нами части Мира как «вакуум», «поля», «тёмная материя», «тёмная энергия» и до сих пор не решили, следует ли к ним применять определение «материальное».

Можно сказать, что «материальный мир» - это основное проявление глобального Мира нам «людям». То есть мы, люди, воспринимаем глобальный Мир в виде относительно объективных для нас, конечных, ограниченных, «материальных» форм, взаимодействующих между собой конечным, ограниченным способом. Мы связываем эти материальные проявления Мира условными, но действительными для нас людей закономерностями, выраженными в наших условных символах.

Воздействие на нас остальной, большей части Мира, которую можно было бы назвать «не совсем материальным миром» или «субматериальным миром» мы можем обнаружить только по тому или иному воздействию на наш материальный мир и никак иначе. Но и это воздействие мы пытаемся представить как каузальное (причинное) воздействие одних материальных форм на другие материальные формы, а «не совсем материальный мир» обычно представляем в виде некоего виртуального посредника между материальными формами. И законы этого «субматериального мира» записываем в виде некоторого в основном вероятностного влияния на материальные формы.

Вот какое определение «субматериального» даёт ChatGPT:

«Для обозначения сущностей, которые являются лишь частично материальными, таких как поля (гравитационное, магнитное и т. д.) и темная материя, которые оказывают влияние на физический мир, но не полностью поддаются непосредственному восприятию, можно использовать термин "субматериальные". Этот термин подразумевает, что данные сущности не вполне материальны в обычном понимании слова, но, тем не менее, они оказывают реальное воздействие на материальный мир и могут быть изучены с помощью научных методов. "Субматериальный" подчеркивает их промежуточное положение между полностью материальными объектами и полностью абстрактными или нематериальными концепциями».

То есть в обычной человеческой практике, да и в науке тоже, мы, как правило, оперируем закономерностями, связывающими между собой «материальные», достаточно хорошо наблюдаемые нами формы Мира и оставляем почти без внимания то, «не совсем материальное», что имеется между ними. И лишь в последние 150 лет, с момента введения (Фарадеем) понятия «электрическое поле», а затем понятий и иных видов полей, и далее понятий «виртуальная частица», «тёмная энергия», «тёмная материя», научное сообщество стало учитывать, что «материальный мир» это только условная часть глобального Мира.

Причём очень важно то, что абсолютной границы между «материальным» и «не совсем материальным» не существует. Критерием «материальности» и «не совсем материальности» является «субъект-человечество» (или в более общем случае «субъект-жизнь»). И обо всём, что существует в нашей локальной части Вселенной, мы можем говорить только с точки зрения наблюдателя «человека», а не с какой-то «абсолютной» внесубъективной точки зрения, как совершенно необоснованно пытались и пытаются делать деятели науки во все времена.

Введением понятий различных «полей» и порождаемых ими «виртуальных частиц» мы уходим от раздражающей ещё Ньютона «концепции дальнодействия» и приходим к некоей «концепции близкодействия». То есть мы вводим в нашу картину Мира различные, практически никак не воспринимаемые, короткоживущие, виртуальные частицы-посредники для объяснения взаимодействия-близкодействия между материальными формами.

Резюмируя можно сказать, что мы, субъекты-люди, в силу ряда неизбежных и неустранимых в принципе сущностных, временных и локальных ограничений, описываем более-менее доступную нам часть Мира удобным, общеупотребимым среди нас, людей, способом. Вводя условные качества и параметры относительно объективных для нас форм и фиксируя некие условные, действительные для нас закономерности их связывающие. Которые, конечно же, субъективны, но, тем не менее, как-то отражают то, что происходит в глобальном Мире и имеет значение для нас, людей.

Но также мы вынуждены признать, что для более полного, в нашем понимании, объяснения Мира, материальных, хорошо наблюдаемых, относительно объективных форм Мира нам недостаточно. И приходится вводить некие условные понятия и для не совсем материальных форм, чтобы более удобно их обозначать в обычной практике и научных гипотезах и теориях.

Мне кажется, что именно введение субъекта-человечества в качестве критерия деления Мира на  «материальное» и «не совсем материальное», и определение «материального» как  наиболее доступной нашему человеческому восприятию части Мира, а «не совсем материального» как гораздо менее доступной для нашего восприятия части Мира, но, тем не менее, существующей и даже поддающейся описанию с применением математических абстракций, могла бы коренным образом изменить всё наше научное мировоззрение.

Любая «причинность» это условная, субъективная конструкция

Итак, на основании всего вышеизложенного можно прийти к следующему выводу – любое, так называемое «объяснение Мира» это условность, по сути «выдуманная» тем или иным субъектом-наблюдателем и существующая только в его специфической системе отсчёта. Но с другой стороны нельзя сказать, что это «объяснение Мира» придумано совершенно на пустом месте и не имеет никаких более глубоких оснований. Если оно вполне сносно работает и помогает субъекту самосохраняться в Мире, то оно по-своему правильно и является его особым, субъективным способом отражения глобальных мировых процессов. Это его собственный уникальный взгляд на Мир.

Если же это объяснение работает и для других субъектов, мы можем считать его «относительно объективным». Но никогда «абсолютно объективным», потому что в Мире существуют и другие субъекты, для которых это объяснение не имеет смысла и никак ими не может быть использовано.

Мир бесконечно сложен и мы никогда не узнаем, что происходит в нём «на самом деле» (если об этом вообще имеет смысл говорить). Так получилось, что мы находимся в более-менее стабильной (временно) части Мира и можем её описать в условных терминах и символах распространённых между нами по некоей общепринятой договорённости.

Тема условной объективности человеческого познания очень важна в философии. Приведу только несколько из многочисленных цитат по этому поводу:

Фрэнсис Бэкон, английский философ, историк (XYII век): «Человеческий  разум по своей склонности легко предполагает в вещах больше порядка и единообразия, чем он их  находит. И в то время как многое в природе единично и совершенно не имеет себе подобия, он придумывает параллели,  соответствия и отношения, которых нет».

Иммануил Кант, немецкий философ (XVIII век): «Однако не предмет заключает в себе связь, которую можно заимствовать  из  него  путем восприятия...а сама связь есть функция рассудка. Этот принцип есть  высшее  основоположение во всем человеческом знании...    Каковы предметы в себе и обособленно от нашей  чувственности, нам совершенно неизвестно. Мы не знаем ничего, кроме свойственного нам способа  воспринимать  их, который к тому же необязателен для всякого существа, хотя и должен быть присущ каждому человеку».

Артур Шопенгауэр, немецкий философ (XIX век): “Всякая  причинность  существует только в рассудке и для рассудка, и, следовательно, весь этот действительный, т.е. действующий мир,  как  таковой,  всегда обусловлен рассудком и без него - ничто.”

Фридрих Ницше, немецкий философ (XIX век): «Мы, одни мы сочинили и причины и последовательности, зависимости, относительность, принуждение, число, закон, свободу, основание, цель; примешивая мир этих знаков, как таковых, существующих "в себе", к вещам, присочиняя его к вещам, мы в очередной  раз  поступаем так,  как поступали всегда, - мы мифологизируем. …

"Видимость” есть прилаженный и упрощённый мир, над которым поработали наши практические инстинкты, он для нас совершенно истинен, а именно: мы живём в нём, мы можем в нём жить,- это есть доказательство его истинности для нас….В основе лежит всегда вопрос “что это для меня ? (для нас, для всего живущего и т.д.) …

Человек вообще не мог бы жить, не допускай он логических фикций, не измеряй он действительность вымышленным миром безусловного, самоотождествленного, не фальсифицируй он (беспрерывно!) мир посредством числа. Истина  есть род заблуждения, без которого определенный род живых существ не мог бы жить. Ценность для жизни  решает в конечном счете».

Освальд Шпенглер, немецкий историософ (XX век): «Каждой культуре присущ уже вполне индивидуальный способ видения  и познания мира-как-природы, или - что одно и то же - у каждой есть своя собственная, своеобразная природа, каковой  в точно таком же виде не может обладать ни один человек иного склада».     

Карл Поппер, австро-британский философ и социолог (ХХ век): «Ни один ряд, скажем, из трех или более причинно связанных конкретных событий не выстраивается в соответствии с каким-то одним законом природы».

Дэниел Деннет, американский философ и когнитивист (ХХ век): «Пространство причин создается в процессе человеческой деятельности, стремления видеть во всем причинно-следственные связи и пронизано нормами, как социально-этическими, так и инструментальными… Приписывание другим неких намерений является характеристической чертой человеческого познания и взаимодействия…

Мы начали с безжизненного мира, в котором не было никаких причин и целей вовсе, но протекали различные процессы: планеты вращались, ледники замерзали и таяли, действовали приливы, извергались вулканы и происходили миллиарды химических реакций. Некоторые из этих процессов порождали новые процессы, которые порождали свои процессы, пока в один прекрасный «момент» (только не воображайте, пожалуйста, что вдруг сверкнула молния, и…) не получилось так, что возникла возможность описать причины, по которым все сложилось так, как оно есть сейчас…

Этот вид деятельности, проверка и оценка причин, движущих нами, не занимает, конечно, каждую свободную минуту, однако играет решающую роль в координации наших действий, в подготовке детей и молодежи к их взрослым ролям, в установке норм, согласно которым мы оцениваем один другого. Эта деятельность играет такую важную роль в существовании нашего вида, что нам порой трудно вообразить, как другие социальные виды – дельфины, волки и шимпанзе, к примеру, – могут обходиться без нее».

Возникновение нашей уникальной Вселенной это цепь случайностей

В предыдущих моих публикациях (например здесь) я утверждал, что любое «нечто» в Мире, начиная от мельчайших частиц и заканчивая Вселенными, появляется по большому счёту случайно, существует некоторое время, а потом неизбежно исчезает. Причём мы не можем рассматривать и изучать это «нечто» в абсолютном смысле, а только в той или иной условной системе отсчёта (связанной с условным субъектом-наблюдателем). В такой системе отсчёта это «нечто» описывается рядом условных параметров относительной стабильности.

Проиллюстрируем всё это на примере последних данных о нашей Вселенной в свете наиболее признаваемых научным сообществом теорий.

Считается, что наша Вселенная возникла около 14 млрд. лет назад буквально из точки в результате так называемого «Большого Взрыва».

В своей книге «Математическая Вселенная» Макс Тегмарк пишет – «Наша Вселенная — это сферическая область с условным центром на планете Земля. Материя у края Вселенной, от которой свет едва успел дойти до нас за 14 млрд. лет, находится сейчас на расстоянии 5 × 1026 м. Насколько сегодня известно, наша Вселенная содержит около 1011 галактик, 1023 звёзд, 1080 протонов и 1089 фотонов… 

На практике любая бесконечная Вселенная может выглядеть извне как чёрная дыра субатомного размера. Пространство расширяется внутрь, не требуя увеличения места, которое оно занимает при наблюдении извне, Эйнштейн позволил пространству растягиваться и порождать дополнительный объём из ничего…

Дальнейшее развитие образовавшейся Вселенной может быть обусловлено множеством случайных факторов. Сейчас известно 32 независимых параметра нашей Вселенной, для которых мы пытаемся измерить как можно больше знаков после запятой».

Макс Тегмарк утверждает, что все эти параметры проходят строгий статистический тест на случайность. И пока что нет теории, которая показала бы нам, что они не случайны. Вариации любого из параметров делали бы нашу Вселенную другой, иногда даже весьма радикально.

К примеру, известно, что наше пространство содержит 3 измерения. Математически доказано, что при числе измерений пространства более трёх, не было бы ни стабильных планетных систем, ни устойчивых атомов. Переход, допустим, в четырёхмерное пространство изменяет ньютоновский закон обратных квадратов для силы гравитации на закон обратных кубов, при котором вообще не существует устойчивых орбит. Пространства с числом измерений менее трёх тоже не позволяют существовать планетным системам, поскольку гравитация в них перестаёт притягивать. Кроме того, они, по-видимому, ещё и по иным причинам слишком просты, чтобы содержать наблюдателей — например, в них отростки двух нейронов не могут пересекаться, не нарушая взаимную целостность.

«Согласно теории струн, истинное пространство всегда имеет 9 измерений, но мы не замечаем 6 из них, поскольку они микроскопически свёрнуты наподобие цилиндра. Если пройти небольшое расстояние вдоль одного из 6 скрытых измерений, окажешься на том же месте, откуда отправился. Предполагается, что все 9 измерений первоначально были свёрнуты, а затем в нашей области космоса инфляция растянула три из них до астрономических размеров, оставив остальные крошечными, невидимыми…

Может ли пространство замёрзнуть? Рыба может думать, что вода — пустое пространство, поскольку это единственная известная ей среда. Но если умная рыба выведет физические законы, управляющие молекулами воды, она поймёт, что у этих уравнений есть три решения: «фазы» жидкой воды, которую она знает, а также пара и льда, которых она никогда не видела.

Этот пример может показаться глупым, и если бы настоящая рыба думала подобным образом, мы могли бы поднять её на смех. Но не может ли быть так, что пространство, которое воспринимается людьми как пустое, также некая форма среды? Тогда будут потешаться над нами. Имеется множество свидетельств того, что так дело и обстоит. Наше «пустое пространство», по-видимому, не только является такого рода средой, но и, похоже, может находиться не в трёх фазах, а в гораздо большем их числе (вероятно, около 10500), а возможно, даже в бесконечном числе. Значит, в дополнение к искривлению, растяжению и вибрации наше пространство, вероятно, способно испытывать нечто подобное замерзанию и испарению» - Макс Тегмарк, «Математическая Вселенная».

Необходимо также отметить, что, скорее всего, даже в нашей Вселенной в различных её областях действуют различные закономерности. «Многие закономерности, которые мы привыкли считать фундаментальными, по определению соблюдающимися всегда и везде, оказались не более чем эффективными законами — локальными нормативными актами, которые могут меняться от места к месту в зависимости от различные фазовых состояния пространства в данной области» - как отмечает там же Макс Тегмарк.

Но пойдём далее, следующие важные параметры нашей Вселенной это:

а) плотность вещества (порядка 10–29 кг/м3) которая обеспечивается тёмной и  барионной материями и благодаря гравитации помогает формироваться галактикам.

б) плотность тёмной энергии (порядка 10–27 кг/м3) которая наоборот противоположна гравитации и способствует расширению Вселенной.

Если бы тёмная энергия имела большую плотность, галактики сформироваться не смогли бы и результатом явилась бы мертворождённая Вселенная, вечно тёмная и безжизненная, не содержащая ничего сложнее почти однородного газа. Если, с другой стороны, плотность тёмной энергии была бы значительно ниже, наша Вселенная прекратила бы расширяться и коллапсировала в Большом Хлопке, прежде чем успела бы появиться жизнь.

Теперь рассмотрим возможные случайные вариации второго из четырёх видов взаимодействий материи в нашей Вселенной - электромагнетизма. Если электромагнитные силы ослабли бы примерно на 4 %, Солнце немедленно взорвалось бы: атомы его водорода стали бы соединяться в дипротоны (не существующую без такой поправки разновидность гелия, не содержащего нейтронов). Если существенно усилить электромагнетизм, то стабильные атомы, например углерод и кислород, будут испытывать радиоактивный распад.

И если бы слабое ядерное взаимодействие (третий вид взаимодействий)  оказалось ещё слабее, то вокруг нас не было бы водорода, поскольку вскоре после Большого взрыва весь он превратился бы в гелий. В обоих случаях — если бы взаимодействие было бы гораздо сильнее или слабее — нейтрино при взрыве сверхновой не могли бы рассеять в космосе внешние слои звезды, и необходимые для жизни тяжёлые элементы вроде железа вряд ли смогли бы покинуть звёзды, где они образуются, и оказаться в составе планет, например Земли.

А что произошло бы в нашей Вселенной, если бы массы элементарных частиц (которые также, скорее всего, случайны) были бы иными? Так, если бы электроны были гораздо легче, то не было бы стабильных звёзд, а если значительно тяжелее, то не могли бы существовать упорядоченные структуры, например кристаллы или молекулы ДНК. Если бы протоны оказались на 0,2 % тяжелее, они превращались бы в нейтроны, неспособные удерживать возле себя электроны, — и не было бы атомов. Напротив, если бы протоны были существенно легче, то нейтроны внутри атомов превращались бы в протоны, так что не было бы устойчивых атомов, кроме водорода. На самом деле масса протона «нуждается» в точной настройке до 33 цифры после запятой, чтобы могли существовать стабильные атомы, кроме водорода.

Наверное, можно сделать следующий вывод – наша Вселенная стала такой, какой мы её наблюдаем, совершенно случайно. Вероятно, существуют и другие Вселенные, которые устроены иначе, чем наша и это тоже произошло случайно. Причём каждая Вселенная со своими уникальными параметрами рано или поздно отживёт свой срок и исчезнет. Но, в течение этого срока Вселенная, с совокупностью всех своих случайных параметров, более менее устойчива и большинство процессов в ней относительно стабильны.

Случайное возникновение самокопирующихся цепочек и «жизни»

Итак, мы предполагаем, что в результате ряда случайных явлений и процессов наша Вселенная приобрела некую конфигурацию. О которой мы не можем сказать ничего конкретного, пока не привязались к определённой системе отсчёта. Ниже мы более подробно поговорим о тех или иных возможных системах отсчёта, а пока лишь укажем на то, что одна из таких систем отсчёта появилась в нашей Вселенной примерно через 14 млрд. лет после её возникновения. И называется она - «человечество».

Нет никаких особых причин считать, что так было задумано кем-то изначально (хотя многим хочется в это верить) поэтому давайте ретроспективно рассмотрим, какие события в нашей Вселенной могли привести к возникновению таких явлений как «жизнь» и «человечество».

Возможно, что большинство существующих в Мире Вселенных мертво и там не сложились условия возникновения некоей субъективности. Но в нашей Вселенной они как-то сложились.

Начнём с того, что по истечении первых нескольких сотен тысяч лет, в которые наша Вселенная была очень горяча и никаких стабильных структур в ней образоваться не могло, некое сочетание сил притяжения (гравитации) и сил расталкивания (воздействие тёмной энергии) о которых мы говорили выше, привело к образованию газопылевых облаков, где впоследствии зародились  галактики и звёзды.

Но все звёзды имеют ограниченный срок своего существования и при его окончании они, так или иначе, распадаются с выбросом в пространство более тяжёлых, чем водород и гелий, химических элементов. И это со временем привело к образованию планет.

На одной из которых произошло практически невероятное событие (но в вечно существующем Мире может сбыться даже самое невероятное) – цепочки некоторых химических элементов «научились» самокопироваться. Со временем эта их способность как-то совершенствовалась, то есть некоторые из вариантов таких цепочек начинали делать свои копии быстрее, чем другие варианты подобных цепочек. Различными способами эти самокопирующиеся цепочки стали распространяться по всей нашей Вселенной и скорее всего их в ней достаточно много.

Попадая в различные условия в тех или иных областях Вселенной самокопирующиеся цепочки видоизменялись и, по крайней мере, на одной из планет приобрели некую углеродную основу. На каком-то этапе первоначально достаточно простые цепочки стали объединяться в более сложные, но также самокопирующиеся структуры. Которые конкурировали между собой за ресурсы для самокопирования и некоторые из них, более совершенные, захватывали всё большую долю в их общей популяции.

Именно эволюция углеродных самокопирующихся цепочек привела к появлению на нашей планете Земля такого явления как «жизнь». Этому способствовало энергетическая стабильность нашей звезды Солнца и определённое расстояние между ней и Землёй, благодаря чему такое важное для самокопирующихся форм химическое соединение как «вода» находится на Земле в основном в жидкой фазе.

Сыграла свою позитивную роль также определённая гравитационная стабильность системы Земля-Луна и наличие неподалеку  громадного Юпитера, который принимает на себя удары большинства странствующих космических глыб. Благодаря металлическому ядру Земля приобрела сильное магнитное поле, защищающее «жизнь» от всеразрушающей космической радиации и способность удерживать довольно плотную атмосферу. В итоге на Земле сформировался примерно 5 километровый слой, в котором самокопирующиеся «живые организмы» чувствовали себя достаточно комфортно.

Опять же нет никаких причин объяснять дальнейшее их усложнение неким божественным или иным замыслом. Всё это лишь порождение конкуренции, случайных ошибок при самокопировании (мутаций) и условий определённой среды, в которой также волею случая оказывались те или иные самокопирующиеся образования. Первым на это указал английский биолог Чарльз Дарвин и назвал процесс изменения живых (самокопирующихся) организмов - «естественным отбором».

Современная версия теории «естественного отбора» хорошо описана в книге Ричарда Докинза «Эгоистичный ген». В которой автор выдвигает гипотезу, что главными в процессе естественного отбора на Земле являются как раз самокопирующиеся цепочки – «гены», а сами организмы это лишь созданные генами «машины» для их распространения (но об этом, более подробно, ниже).

На всём протяжении истории Земли новообразованных субъектов – «живых существ» поджидало ещё немало непредсказуемых поворотов судьбы, которые значительно повлияли на их внешний вид и способы функционирования. Это и сильная периодами вулканическая активность и всё же достигающие Земли большие метеориты, меняющаяся мощность излучения Солнца и многие другие случайные явления и процессы в их комплексе. В итоге археологи наблюдают при раскопках смену климатических эпох в прошлом, странные периоды резкого видообразования живых организмов или их исчезновения. Живые существа на Земле принимали различные формы, но основная их сущность – самокопирование при использовании окружающих ресурсов и конкуренция за них оставалась неизменной.

Возникновение человека, речи, сознания, цивилизации также случайно

Карл Поппер, австро-британский философ и социолог (ХХ век): “Что касается теории Эволюции, то как мне кажется с полным основанием можно сказать только следующее: общая тенденция к увеличению  разнообразия форм  и  т.  п.  позволяет утверждать, что "прогресс" иногда происходит, а иногда -  нет,  что  эволюция  некоторых  форм иногда прогрессивна, в большинстве же случаев - нет, и что у нас нет общего основания ожидать в будущем  появления  форм, способных к дальнейшему прогрессу”.

Я убеждён, что эволюция живых существ не имеет какой-то генеральной линии на всё возрастающее усложнение. Большинство населяющих Землю живых организмов достаточно примитивны и в таком примитивном (с нашей точки зрения) виде существуют уже миллионы и миллиарды лет. Тем не менее, некоторые из них становились всё более сложно устроенными.

Одно из таких усложнений - это появление так называемой «нервной системы», которая позволила некоторым живым существам управлять своим телом более эффективно. И приобрести, в связи с этим, большую подвижность и скорость реакций, более широкий обзор и что самое важное, умение накапливать некоторый индивидуальный, а не только видовой опыт. В дальнейшем нервная система некоторых организмов становилась всё более централизованной, что ещё более способствовало развитию самоориентации живых существ.

Современных людей от приматов отличает не только прямохождение, но,  главным образом, наличие речи, письменности, более сложных способов предвидения и построения социума. Хотя никаких особых отличий в генетике у нас с приматами нет. Вполне возможно, что человечество могло и вовсе никогда не появиться, а происхождение речи до сих пор остаётся загадкой для исследователей (хотя нет недостатка в различных версиях). Предположительно элементарные способности к речи существовали у гоминидов на протяжении миллионов лет, но еще 50 тысяч лет назад язык был в лучшем случае рудиментарным.

Многие из антропологов считают, что используемый нами речевой аппарат не возник специально именно для речи. Далёкие наши предки, как и мы, просто использовали для этого «уже готовую» гортань, изначально приспособленную к дыханию, глотанию, иммуннозащитным функциям, что для человеческой особи гораздо важнее, чем речь. В частности подмечено, что все мировые языки базируются примерно на 90 звуках, т. е. на ничтожно малом количестве, по сравнению с тем необыкновенным множеством звуков, которые человек способен распознать.

 Объяснять наличие речи и значительно более развитого сознания у человека (по сравнению с приматами) размерами мозга и какими-то особенностями его анатомии, необычной электрохимической активностью нервной системы наверное нельзя. Скорее всего, этому способствовал некий комплекс явлений (в том числе проживание в общине, усложнение деятельности) который вдруг дал людям возможность заговорить, поначалу с трудом, а потом все более совершенно и затем это вылилось в более быстрое развитие сознания.

Кстати вариации размеров мозга у современных людей от 400 грамм до 2500 грамм почти никак не влияют на интеллект. В основном мозг и центральная нервная система обслуживают другие, более «примитивные» потребности нашего тела: восприятие мира, движение, обмен веществ, выработку условных рефлексов.

Необходимо также отметить, что на протяжении почти 2 миллионов лет наши предки, имея развитый мозг, равный мозгу Эйнштейна не придумали ничего сложнее каменных ножей и примитивных приспособлений для охоты. И довольствовались ролью мелкостайного животного, промышлявшего в основном падалью и выкапыванием съедобных корешков.

Но примерно 40 тысяч лет назад эволюция человека вдруг пошла ускоренными темпами и 30 тысяч лет спустя уже возникла первая цивилизация шумеров (на территории Ирака), а потом и другие, ещё более сложно устроенные сообщества, придумавшие арифметику, календарь, часы, колесо (правда не все) металлургию, различные способы земледелия, одомашнивания животных, строительства, передвижения по суше и морю. Ещё через 10 тысяч лет человек начал запускать космические корабли и разрабатывать искусственный интеллект.

Многие исследователи считают всё произошедшее тогда (40 тысяч лет назад) с человечеством некоей аномалией и совсем не обязательной случайностью. Появление и развитие речи, а потом и связанное с этим значительное усложнение сознания могло произойти значительно раньше, могло произойти позже, а могло и вообще не произойти.

Как на один из факторов, способствовавших эволюции первобытного человека, многие антропологи указывают на его особую агрессивность. Которая, при отсутствии мощных клыков и когтей, как-то помогала первым людям в освоении мира. Воинственность и агрессивность нашего вида проявлялась и в дальнейшем на протяжении всей истории, где в почёте всегда были воины-завоеватели. Современным людям надо научиться снижать эту свою избыточную агрессивность. Потому что при наличии сверхмощного оружия человечество запросто может самоуничтожиться, превратив Землю в ад, в котором выживут лишь более уравновешенные тихоходки, не имеющие ни интеллекта, ни научных методов познания мира.

И, конечно же, нет никаких оснований считать современного человека с множеством анатомических и физиологических недостатков неким окончательным, совершенным вариантом эволюции. К примеру, в конструкцию глаза хаотично «вложились» эпохи начиная с раннего кембрия, обеспечив наш глаз набором конструкционных нелепостей, вроде вывернутой наизнанку сетчатки или слепого пятна. Своеобразно охарактеризовал странности глаза человека Тревор Лэм из австралийского центра наук о зрении: «Фактически, если бы глаз со всеми его недостатками спроектировали инженеры, они были бы уволены».

Скорее всего, организм людей (если они всё-таки выживут) будет изменяться и в дальнейшем, возможно даже с применением искусственных элементов.

Возникновение и распространение репликаторов нового типа - Мемов

Итак, следуя гипотезе, предложенной известным современным эволюционистом Ричардом Докинзом, мы будем считать главным признаком человека, как и всех прочих живых существ на Земле - способность обнаруживать, поглощать и перерабатывать вещества, содержащие преимущественно углерод, кислород, водород, азот, кальций и фосфор для «производства» копий формирующих их организмы репликаторов-генов (а также генных проявлений) и дальнейшее распространение этих генов вовне путём размножения.

Но оказывается некоторое время назад на Земле появились репликаторы иной, нежели гены природы – так называемые «мемы».

Вот как это описывает сам Ричард Докинз в своей знаменитой кнгиге «Эгоистичный Ген» – «Основная единица жизни, её главный двигатель - это репликатор. Репликатором можно назвать любой объект во Вселенной который самокопируется. Репликаторы появляются главным образом случайно, в результате беспорядочного столкновения мелких частиц. Однажды возникнув, репликатор способен генерировать бесконечно большое множество собственных копий. Случилось так, что реплицирующейся единицей, преобладающей на нашей планете, оказался ген - молекула ДНК”.

В течении более чем трёх миллиардов лет ДНК была единственным на свете репликатором, заслуживающим внимания. Однако она не обязательно должна сохранять свои монопольные права навечно. Всякий раз, когда возникают условия, в которых какой либо новый репликатор может создавать собственные копии, эти новые репликаторы будут стремиться взять верх и начать собственную эволюцию нового типа.

Мне думается, что репликатор нового типа недавно возник именно на нашей планете. ... После появления самокопирующихся мемов началась их собственная, гораздо более быстрая эволюция. … Язык - это всего лишь один из многих примеров. Мода на одежду и еду, обряды и обычаи, искусство и архитектура, техника и технология - всё это развивается в историческое время, причём развитие это напоминает сильно ускоренную генетическую эволюцию, не имея на самом деле никакого к ней отношения».

То есть «мем», согласно Докинзу, это некая, способная и стремящаяся реплицироваться условная форма (явление, особенность), берущая начало и существующая, прежде всего, в психической системе некоторых живых существ, но не имеющая прямой связи с генами. И обычно к мемам относят элементы поведения, внешности и мышления,  которые возникают и передаются  не генетическим, а иным путём, а именно посредством наблюдения и общения между живыми существами.

Конечно, сама способность живого копировать элементы поведения, внешности и мышления как свои собственные, так и своих современников есть результат генетических программ, и передаётся посредством генов. Но, несколько упрощая, можно от генов как бы отстраниться и утверждать, что в среде наиболее продвинутых живых существ  возникают и распространяются так называемые «мемы»,  которые также как и гены оказывают весьма  небескорыстное  влияние на организм. То есть у мемов, как и у генов, достаточно хорошо прослеживается «эгоистичное стремление» самосохраниться и самораспространиться, используя организм лишь как временную базу и «устройство» для своего выживания и распространения.

То есть мем может как привходить в живое существо извне, так и возникать в нём самостоятельно. Оказавшись в организме, мем стремится распространиться (копироваться) в нём и это выражается в том, что организм начинает повторять сам за собой определённые действия (или прокручивает одни и те же желания, мысли). В этом случае обычно мы говорим, что некое действие вошло в привычку (или мысль стала естественной или навязчивой). Но можно также сказать, что образовавшийся в организме или вошедший в него мем  просто стремится копировать самого себя.

Несомненно самой возможности возникновения таких самореплицирующихся форм как мемы способствовало эволюционное развитие психической системы (и мозга, как её части) живых существ. У более примитивных организмов мемы невозможны. Таким образом, для существования и распространения мемов нужна особая среда – развитые, сложно устроенные психические системы. И немалую роль в этом играют так называемые «зеркальные нейроны».

В последние сто лет на помощь мемам пришла техника. И мемы стали не только передаваться при прямом общении людей, но и реплицироваться посредством печати, радио, кино и телевидения, которые, по сути, распространяют и навязывают читателям, слушателям и зрителям те или иные мемы - образцы поведения, внешности и мышления.

И, надо заметить, мемы всё больше и больше определяют жизнь людей, часто вторгаясь и в область влияния генов. Например, гены «обязывают» человека искать полового партнёра и плодить потомков, а мем «обета безбрачия» или «полового воздержания» хотя и с большим трудом, но подавляет эти стремления. Гены заставляют организм регулярно потреблять пищу, а мем «долга или взаимопомощи» может принудить человека отказаться от неё в пользу существа даже и не содержащего гены данного организма. Гены требуют от индивида самосохранять и защищать себя, а мем какой-то «идеи», «веры» или «протеста» способен подвигнуть человека на смерть ради сохранения и распространения всего лишь данного мема.

Причём следует понимать, что мему - привычке, образу мышления или жизни и, в некоторой степени, техническому устройству, в общем-то «безразлична» судьба своего основного носителя – человека. Ему главное самому реплицироваться, а что при этом будет с его временным пристанищем (организмом), как правило,  всё равно.

Высокий иерархический ранг позволяет особи - индивиду успешнее распространять свои гены и мемы

Как показывают наблюдения, организмы, проживающие стадом (стаей, роем, группой) не так уж часто серьёзно конфликтуют друг с другом. Наверное, в противном случае, у них на всё остальное не оставалось бы ни времени, ни сил и они постоянно страдали бы от ран наносимых друг другу в результате выяснения отношений. Обычно в сообществах живых существ имеются некие правила (и, соответственно, признаки, сигналы) по которым особи определяют - кто из них потенциально сильнее, ловчее и так далее. Лишь изредка, а именно в период завоевания полового партнёра, они схватываются почти по-настоящему, так как размножение (а по сути - распространение своих условных частей – генов) это то основное, ради чего они вообще существуют.

Ведь у большинства видов организмов к размножению допускаются только выдающиеся особи, сумевшие выстоять в конкурентной борьбе и подняться на самый верх иерархической лестницы. Особи противоположного пола, также стремящиеся размножиться (распространить свои гены),  наблюдая за этой конкуренцией, по ряду признаков определяют иерархов и спариваются именно с ними, что в итоге даёт более сильное и приспособленное потомство.

Говоря иначе, любая особь стремится пристроить свои условные части - гены  туда,  где существует наибольшая вероятность к их дальнейшему распространению. Если же смотреть на всё это с позиции генов, то они, стремясь реплицироваться, «выискивают» те особи, которые смогут сделать это наиболее лучшим образом и встраиваются в них.

По сути дела все мы потомки иерархов и, следовательно, стремление подниматься вверх по иерархической лестнице заложено в нас изначально (генетически). Правда в большинстве людских сообществ основное иерархическое деление (основанное на размерах особи-индивида и его силе) несколько смягчено и, в частности, к размножению «допускаются» индивиды не являющиеся безусловными иерархами. Но, тем не менее, возможность выбирать себе партнёра, а также условия жизни и забота о потомстве во многом  определяются местом человека на социальной иерархической лестнице.

Укажем ещё на то, что среди людей также в ходу определённые правила (признаки), по которым они определяют принадлежность  индивида-личности к той или иной ступени иерархии. Но одно из отличий человека от животных заключается как раз в том,  что в обществе обычно существует не одна, а несколько иерархических лестниц. И различные группы людей признают помимо основных, ещё и некоторые  специфические виды  иерархий.

Как животное,  стоящее рангом ниже, подчиняется (как правило) воле иерарха, так и индивид, занимающий   более низкую ступень иерархической лестницы, подчиняется (обычно) требованиям вышестоящего лица. Так как миллионолетняя практика показала, что неподчинение может привести к довольно тяжким последствиям для неповинующегося. Но, тем не менее, такая зависимость и страх не являются большим препятствием для обмана, воровства, подсиживания как в среде животных, так и в людских сообществах.

Что же всё-таки даёт особи - индивиду его более высокий иерархический ранг?  Очевидно, что и иерарх-животное и иерарх-человек имеют большие, по сравнению с остальными, возможности в использовании окружающей среды и обладают более лучшим половым партнёром - распространителем его генов. Иерархи имеют также  возможность навязывать другим свою волю, а в случае людей – ещё и мнения, идеи. Короче говоря, высокий иерархический ранг индивида предоставляет как его генам, так и  мемам  более лучшие условия для самосохранения и распространения.

То есть, чем выше ранг человека, тем больше у него возможностей для распространения условных форм (генов и мемов), из которых он сам и состоит. А, следовательно, «власть» - это даже не столько перспектива успешно распространить свои гены, сколько прекрасная возможность распространять свои собственные мемы. Ведь подчинённые только и делают, что подстраивают своё поведение под поведение своего шефа и исполняют пришедшие ему в голову идеи. Заметим, что представители творческих и общественных профессий - политики, артисты, учёные тоже более озабочены распространением  мемов, а не генов.

Мы, обычные люди, также всю жизнь вглядываемся в тех, кто находится выше нас на иерархической лестнице и перенимаем, копируем их привычки, внешность, стиль, образ мышления и жизни. Потому что (и отчасти  справедливо) полагаем, что именно эти мемы  и  помогли им подняться так высоко.

Как правильно описывать Вселенные

Согласно «инфляционной модели» - фаворита из современных космологических моделей, Вселенные в глобальном Мире возникают из точек квантовых размеров. То есть, по сути, надуваются как воздушные шарики практически из ничего, благодаря бесконечной энергии имеющейся в Мире. Причём сам процесс развития той или иной конкретной Вселенной мало предсказуем, так как зависит от начальных условий, задаваемых неопределённостью на квантовом уровне. Каждая Вселенная особенна по-своему и характеризуется рядом параметров, которые, так или иначе, влияют на дальнейшее её существование.

Как уже говорилось выше, скорее всего большая часть Вселенных мертва. Но в каких-то из них создаются определённые условия для возникновения некоей «субъективности». И если «извне» такая Вселенная может по-прежнему «казаться» точкой квантовых размеров, то для новоявленных субъектов их родная Вселенная изнутри может «выглядеть» практически бесконечной. Хотя любой субъект, возникший в этой Вселенной, ограничен в своём восприятии и воспринимает её в виде условных, конечных, ограниченных форм, взаимодействующих между собой условным, конечным, ограниченным образом.

Каждый субъект уникален в восприятии им своей родной Вселенной и одни её части воспринимает, скажем так, «более отчётливо», а другие «менее отчётливо». Когда субъектов несколько, они, общаясь между собой, формируют некую общую для них и поэтому «относительно объективную реальность» и могут вводить условные символы для  обозначения тех или иных её условных частей.

К примеру, субъект-человек, как мы отмечали ранее, воспринимает свою Вселенную в виде совокупности материальных форм (объектов, явлений, процессов), которые для него существуют «достаточно отчётливо», а также в виде субматериальных сущностей – полей, тёмной материи и тёмной энергии, существование которых он обнаруживает лишь косвенно, по некоторому влиянию на материальные формы. Люди способны фиксировать те или иные, общие для них, закономерности для наиболее хорошо воспринимаемых ими материальных форм и называют их «законами природы». А закономерности для субматериальных сущностей в основном носят вероятностный характер и описываются «волновыми функциями».

Очень важно то, что на протяжении некоторого периода многие процессы во Вселенной более-менее стабильны и поэтому так называемые «законы природы» это в основном законы сохранения тех или иных условных параметров материальных форм в условно замкнутых системах, существующих и таковых в определённых системах отсчёта.

Одна из задач науки – поиск и описание таких параметров материального мира, которые более стабильны при различных взаимодействиях по сравнению с другими параметрами. Всё в Мире меняется, но одно меняется медленнее, чем другое и может считаться относительно постоянным в течение какого-то периода в какой-то области пространства.

Необходимо также отметить, что любые субъекты не бесконечно чувствительные существа, и, к примеру, мы, субъекты-люди не способны воспринимать отдельные сверхбыстрые и микроскопические явления квантового мира. И поэтому можно сказать, что всё наше восприятие Вселенной это нечто усреднённое по времени и локальности. Все наши параметры для описания явлений и процессов это макропараметры, весьма грубое приближение. И, как следствие, все наши закономерности, скорее всего, не существуют на микроуровне, а являются результатом этого нашего ограниченного, грубого, усреднённого восприятия. Но для нас, субъектов, всё это приблизительное, ограниченное восприятие имеет решающее значение, мы существуем в окружении этих усреднённых приближений, они есть наша реальность.

Например, в первом приближении такой условный параметр как «масса» в большинстве окружающих нас процессов и явления сохраняется. Хотя в микромире, и, наоборот, в глобальных космологических процессах, а также при движении объектов на больших скоростях это не совсем так.

В нашем «приблизительном, условном, ограниченном человеческом мире» скорость передачи любых взаимодействий также ограничена и составляет примерно 300 тыс.км/с. Но, как недавно выяснилось, в системах, так называемых «запутанных объектов» этот принцип не выполняется и при их описании более важным считается принцип сохранения этими системами некоторых своих параметров при внешнем воздействии.

Таким образом, на мой взгляд, правильное описание Вселенной должно быть следующим:

1) Необходимо выбрать систему отсчёта, то есть субъекта, относительно которого и для которого это описание создаётся. Описание Вселенной без указания субъекта не имеет смысла, это всё равно как измерять расстояния не указывая первоначальной точки отсчёта.

2) Важно учитывать, что любой субъект ограничен в своём восприятии Вселенной. Он воспринимает её в виде конечных, ограниченных форм, взаимодействующих между собой конечным ограниченным способом. Причём одни формы он воспринимает «более отчётливо», а другие «менее отчётливо». Он также не является бесконечно чувствительным существом и воспринимает предметы, явления и процессы, происходящие во Вселенной в некоем усреднённом варианте.

3) Если сама Вселенная и возникший в ней субъект существуют относительно долго, это означает, что большая часть процессов во Вселенной также относительно стабильно (всё, безусловно, меняется, но с различными скоростями). Субъект может зафиксировать некие параметры этих процессов (в своей субъективной системе отсчёта), остающиеся относительно стабильными. Причём он должен понимать, что сам факт существования именно этих параметров во многом случаен, то есть задаётся случайными событиями при зарождении Вселенной или произошедшими позже, в процессе её развития. Но именно такие параметры и привели в итоге к появлению самого данного субъекта в этой Вселенной.

4) И наконец, субъект должен учитывать, что в различных частях Вселенной и в различные периоды её существования эти параметры относительной стабильности происходящих в ней процессов могут быть различными.

Условный наблюдатель «человек» в поиске абсолютной системы отсчёта

Джон Сёрл, американский философ (ХХ век): «Вычислительную модель физического процесса никоим образом не следует отождествлять с самим процессом, происходящим в действительности».

Имре Локатос, англо-венгерский философ (XX век): «Логическая теория математики такая же увлекательная, изощрённая спекуляция, как и любая научная теория».  

Большинство исследователей по умолчанию предполагают, что они обладают некоей абсолютно объективной системой отсчёта и большинство их выводов также абсолютно объективно. На мой взгляд это не так. Абсолютно объективных систем отсчёта не существует. Любая система отсчёта субъективна и все, кто исследуют Мир, обязаны это учитывать.

Даже математика – царица наук, не может претендовать на абсолютную объективность. Любое символическое описание мира в виде слов, чисел и пр. всегда условно, субъективно.

Желание обладать какой-то абсолютной и внесубъективной системой отсчёта всегда было присуще человечеству. И в этом учёные материалисты смыкаются с религиозными верующими. Если в религии такой абсолютной системой отсчёта является Бог, относительно которого всё и существует в своём истинном варианте, то в материализме критерием абсолютности и истинности является бесконечно далёкий будущий всёзнающий субъект-человечество. То есть, выдвигая любую гипотезу, мы оставляем окончательное решение о её правильности или неправильности на суд потомков. Которые, вполне очевидно, будут умнее и мудрее чем мы, а также будут обладать более совершенными приборами и методами исследования и обязательно окончательно во всём разберутся.

Но что же на самом деле представляет собой субъект и наблюдатель - «человек»? Организм «человека» состоит из более чем 30 триллионов собственных клеток и не менее чем 40 триллионов бактерий, которые в свою очередь состоят из микрочастиц, положение и движение которые, как утверждает квантовая механика, не определяются точно. Поэтому, на мой взгляд, правильнее было бы считать «человека» некоей условностью, которая воспринимает Мир также условно своим особенным способом и восприятие которого почему-то примерно похоже на восприятие других людей.

То есть, говоря о существовании любого «нечто» в нашем мире, начиная от микрочастицы и заканчивая Вселенной, мы всегда должны указывать на то, что мы определяем это «нечто» в нашей условной человеческой системе отсчёта, используя доступные нам способы восприятия. Мы справедливо считаем это «нечто» субъективным, если оно воспринимается только одним каким-то человеком. И говорим о том, что это «нечто» объективно, если оно воспринимается примерно одинаково всеми остальными людьми. Но я считаю это лишь относительной объективностью, а не абсолютной объективностью.

По моему мнению, относительную общечеловеческую объективность большинство прошлых и современных деятелей науки пытается выдать за некую абсолютную объективность. И в этом их ошибка. Хотя в какой-то степени такое допущение вполне оправдано и может довольно сносно работать на благо всего человечества. То есть мы условно можем считать наиболее продвинутую часть «человечества» - «научное сообщество», максимально объективной системой отсчёта, так как ничего лучшего нет и быть не может для нас людей в принципе.

Резюме

  1. Наша Вселенная и мы сами в ней появились совершенно случайно. Существование человечества обусловлено рядом уникальных, случайных параметров и процессов нашей Вселенной и при других условиях нас бы не существовало или мы были бы совсем другими.

  2. Вселенных в Мире, скорее всего, бесконечно много и каждая может характеризоваться, в той или иной системе отсчёта, некими уникальными параметрами относительной стабильности происходящих в ней процессов, которые мы называем «законами природы».

  3. Некоторые Вселенные на определённом этапе могут порождать в себе субъектов (наблюдателей). Абсолютных наблюдателей не существует. Любой наблюдатель это условность, своеобразная точка обзора Вселенной (Мира) и связанная с ним система отсчёта.

  4. Субъекты не способны генерировать абсолютно объективное знание, а только относительно объективное, действительное для ограниченной группы обитающих в данной Вселенной субъектов, с основной целью - продление своего существования. Субъекты связывают наиболее «отчётливые» для них проявления Мира условными закономерностями, а для «не отчётливых» проявлений используют вероятностные способы описания.

  5. Восприятие Мира субъектами уникально и не всегда доступно даже для относительно объективного изучения. В частности субъекты-живые существа обладают таким своеобразным способом взаимодействия с Миром как «сознание», которое также не может быть изучено даже относительно объективно полностью, так как уникально для каждого живого организма.

Александр Коробов, физик, философ       al.korobov.nd@gmail.com

Комментарии (87)


  1. Tzimie
    23.04.2024 19:00
    +2

    По первому пункту. Под вселенной вы имеете в виду нашу конкретную вселенную пузырь с определенными физическими константами, или, грубо говоря, multiverse, набор всех возможных вселенных?


    1. AleksandrTallinn Автор
      23.04.2024 19:00

      По первому пункту. Под вселенной вы имеете в виду нашу конкретную вселенную пузырь с определенными физическими константами, или, грубо говоря, multiverse, набор всех возможных вселенных?

      Нашу конкретную Вселенную, с нашим набором констант, распределением пропорций между веществом и тёмной энергией и т.д. Там дальше в тексте это указано


      1. Tzimie
        23.04.2024 19:00
        +3

        Тогда согласен. Однако, хотя наша вселенная - случайность, но существование life friendly universe в multiverse - неизбежность

        Это как лотерея, выигрыш для участника случаен, но кто-то выиграет обязательно


        1. AleksandrTallinn Автор
          23.04.2024 19:00

          хотя наша вселенная - случайность, но существование life friendly universe в multiverse - неизбежность Это как лотерея, выигрыш для участника случаен, но кто-то выиграет обязательно

          Да, человечество вытянуло счастливый билет. В нашей Вселенной случайным образом сложились условия для возникновения субъектности. Предположительно большинство Вселенных в Multiverse мертво, а в те Вселенные где другие какие-то субъекты возможно есть нам никогда не попасть


          1. Tiriet
            23.04.2024 19:00
            +9

            это, вообще-то, совершенно плохо обоснованные теории пока- рассуждения про мультивселенные, про "случайность" или "неслучайность" чего-либо в нашей Вселенной, и про счастливый билет. Мы не понимаем, как оно устроено, и потому пока только фантазируем о том, как оно могло бы быть устроено. И в связи с этим очень странно видеть в статье по субьективность наблюдаемой физической реальности такие безапелляционные утверждения о том, что в нашей Вселенной что-то там сложилось "случайным образом".

            А уж вынесение этого положения в выводы- вообще в корне противоречит всему содержанию статьи. По содержанию у Вас все субъективно и зависит от наблюдателя, а в выводах- внезапно- как всеобщий факт подается грязная инсинуация.

            Вы утверждаете, что Вселенных в Мире что-то там. Я, как физик, читаю Вас и задаюсь вопросом- а Вселенная это что? Мир- это что? определение терминов где? нету? тогда, получается, что вся статья- голимая философия. потому что для предмета обсуждения надо сначала этот предмет идентифицировать, а Вы мне про какую-то неизвестную субстанцию рассказываете, которую Вы называете Вселенной и Миром, но только они к моим-то терминам вообще никакого отношения не имеют, получается. как "утка", которая не то крякает, не то в газете.

            Вы рассуждаете про "абсолютное знание". А я опять задаюсь вопросом- "абсолютное"- это что за характеристика?

            Вы рассуждаете про "объективное знание". и опять вопрос- что значит "объективное"? у меня, как у физика, объективное- это которое проверяемое другими исследователями и после проверки сохраняющее формулировки. Вот есть у меня закон сохранения импульса- и он у меня объективный, потому что другие его проверяют- на шариках там, или в соударениях микрочастиц- и тоже приходят к такой же формулировке- импульс в замкнутой системе сохраняется. И мы с этими другими в этой формулировке абсолютно одинаково понимаем все термины, вплоть до массы, расстояния и времени (а массу и время и расстояние мы уже не понимаем, но Вы до туда даже не дошли еще). А у Вас "объективное"- это че? Вы об чем? А оно у Вас в выводах ведь стоит.

            И все Ваши выводы для меня выглядят как бессмысленные. то есть, ассоциативные связи между терминами в этих выводах я вижу, а вот логические связи- отсутствуют, потому что термины в этих выводах не имеют определений, а потому и логических связей между ними нет и быть не может- цепляться связям не за что. Но прочитал с интересом. как пример другого в корне , не связного и поверхностного мышления. прикольно. спасибо за развлечение.


            1. AleksandrTallinn Автор
              23.04.2024 19:00

              Мы не понимаем, как оно устроено, и потому пока только фантазируем о том, как оно могло бы быть устроено

              Согласен с вами, да мы постоянно фантазируем по поводу всего происходящего вокруг, но иногда эти фантазии бывают полезными и мы начинаем их придерживаться называя "законами природы".

              Об этом писал и Ницше

              Фридрих Ницше «Мы, одни мы сочинили и причины и последовательности, зависимости, относительность, принуждение, число, закон, свободу, основание, цель; примешивая мир этих знаков, как таковых, существующих "в себе", к вещам, присочиняя его к вещам, мы в очередной  раз  поступаем так,  как поступали всегда, - мы мифологизируем. …


              1. Tiriet
                23.04.2024 19:00

                Если Вы поддерживаете эту мысль (про фантазии о мультивселенных)- то вдвойне странно видеть ее отголоски в выводах статьи про объективное, абсолютное и суъективное знания.

                Ницше писал. да. Но все, что писал Ницше- с моей точки зрения- надо очень пристально и внимательно просеивать, так как результаты мышления Ницше (сумасшествие) экспериментально показали, что в его ходе мышления что-то явно было неправильным. Понимаете, если у меня есть очень хорошие и грамотные чертежи ракеты, которая после сборки не полетела- то значит, в чертежах ошибка. А если я ее не вижу явно и четко- то это просто значит, что я ее не вижу. И ее надо найти. Так вот с Ницше для меня похоже- раз его мышление привело его туда, куда привело- то оно однозначно ошибочное, надо или разобраться- в чем именно, или вообще "перечертить" все заново, без оглядки на "косячные чертежи".


                1. AleksandrTallinn Автор
                  23.04.2024 19:00

                  Если Вы поддерживаете эту мысль (про фантазии о мультивселенных)- то вдвойне странно видеть ее отголоски в выводах статьи про объективное, абсолютное и суъективное знания

                  Согласитесь, что все наши слова, определения, это наш человеческий способ общения между собой по некоей условной договорённости.

                  Для огромного большинства других субъектов в мире, в том числе и для обитающих на земле животных они не означают ничего.

                  Но для нас людей они имеют определённый смысл. И если благодаря в том числе использованию тех или иных способов общения, мы люди более-менее успешно существуем в мире, значит в этом есть некая условная объективность, общая для нас людей.

                  Мы пытаемся для себя как то объяснять происходящее вокруг в наших субъективных, людских терминах, и это на удивление работает для нашей общей пользы.

                  Кстати это одна из важнейших математических проблем - почему мир вообще можно описать математически и иногда даже слишком просто!


                  1. Tiriet
                    23.04.2024 19:00
                    +2

                    не соглашусь. я считаю, что у нас есть некоторая объективная реальность- например, силикатный кирпич. я также считаю, что у нас есть некоторые объективные абстрактные смыслы- например ноль, единица, число Пи и мнимая единица. Я считаю, что у нас для обозначения этих элементов объективной реальности и этих объективных абстрактных смыслов есть слова. еще раз повторю идею- слова- это способ обозначения объективных "штук" (возможно- не все слова, кстати, выражают объективные штуки, но некоторые- точно). не "для нас имеют смысл", а мы этими словами смысл выражаем! а он есть сам по себе, без нас. и без Вас. наши слова для животных "ничего не означают", но это не значит, что для животных отсутствуют эти смыслы. Котлека имеет смысл и для меня, и для собачки, хотя слово "котлетка" для собачки и не означает ничего, а просто воспринимается как для меня ее гавгавгав. нелепый набор звуков.

                    да, мы некоторые вещи объясняем себе в субьективных людских терминах. но мы специально изобрели математику и физику, чтобы иметь еще и объективные термины. и даже разобрались, когда термин является объективным, а когда- нет.

                    и все это никакого отношения к вопросу о том, почему математика так хорошо описывает реальность- не имеет. :-)


                    1. AleksandrTallinn Автор
                      23.04.2024 19:00

                      да, мы некоторые вещи объясняем себе в субьективных людских терминах. но мы специально изобрели математику и физику, чтобы иметь еще и объективные термины. и даже разобрались, когда термин является объективным, а когда- нет

                      Мы с вами пытаемся столкнуть между собой две философские концепции - материализм и субъективный идеализм.

                      Материализм предполагает что существует некая абсолютная для всех объективность, которую мы обозначаем как то субъективно в наших общечеловеческих терминах.

                      Субъективный идеализм, в той его части которой я придерживаюсь, считает что даже материя (та же "котлетка") существует лишь субъективно, но для некоторых субъектов она может быть относительно объективной, то есть восприниматься ими примерно одинаково. Это есть наш общий удобный для нас способ восприятия мира.

                      И это вполне для нас работает.

                      То есть я считаю, что в мире существуют и другие субъекты (к приаеру в других Вселенных) которые воспринимают мир совсем иначе чем мы и наша объективность для них ничто. И скорее всего они нас никак не воспринимают как и мы их.

                      Вы же считаете, что человек и человечество это некая абсолютная система отсчёта в мире и если что то относительно нас существует, то оно существует и относительно любых других субъектов в мире. С этим я не согасен.

                      Вокруг нас масса примеров относительной объективности. Многие люди видят красный свет светофора, а дальтоники (и многие животные) нет. Значит красный цвет лишь относительно объективен.

                      То же самое можно сказать про котлетку и про материю вообще.


                      1. Tiriet
                        23.04.2024 19:00
                        +2

                        тогда сформулируйте, что значит термин "существует". у меня- существует если воздействует на измерительные приборы и вызывает изменения показаний этих приборов. любых приборов любой природы. хоть колбочки в глазу, хоть весы дорожные на 50 тонн, хоть детекторы сверхтонких чегоугодно. а у Вас "существует"- это что?


                      1. AleksandrTallinn Автор
                        23.04.2024 19:00

                         у Вас "существует"- это что?

                        По моему мнению, любому субъекту Мир будет представлен в виде некоторого количества условных, конечных, ограниченных сущностей (форм) действительных (существующих) для него и взаимодействующих между собой условным, конечным, ограниченным образом.

                        Мы будем называть сущность (форму) «действительной, существующей» для данного субъекта, если она, как таковая, может воздействовать на этого субъекта и, так или иначе, изменять его.

                        Совокупность всех этих действительных для данного субъекта сущностей (форм) Мира мы будем называть «Представлением Мира» данного субъекта, позаимствовав это определение у субъективного идеалиста Артура Шопенгауэра.

                        “Внешний мир в пространстве и времени, проявляющий  себя как чистую причинность, совершенно реален; он и есть, безусловно, то, за что он себя выдает, а выдает он себя всецело и без остатка за представление, связанное по закону причинности. Но, с  другой  стороны,  всякая  причинность  существует только в рассудке и для рассудка, и следовательно, весь этот действительный, т.е. действующий мир,  как  таковой,  всегда обусловлен рассудком и без него - ничто.” -  Артур Шопенгауэр.     

                        Более подробно здесь        


                      1. Tiriet
                        23.04.2024 19:00
                        +2

                        ну вот и славно. "мы будем называть бла-бла-бла существующей бла-бла-бла если она может воздействовать и изменять бла-бла-бла".

                        Материализм в чистом виде. но, что характерно- шизофренический несколько, потому что Вы в конце сформулировали определение термина (материалистическое) а вначале использовали его как Ваш идеалистический. явно продемонстрировав раздвоение суждений, правда- только после того, как я Вас прямым вопросом заставил формулировочку привести. :-). с моей точки зрения, надо было сначала определить термины, а потом использовать их согласно определению, а не так, как получилось :-)

                        С Вами интересно!


                      1. AleksandrTallinn Автор
                        23.04.2024 19:00

                         Вы в конце сформулировали определение термина (материалистическое) а вначале использовали его как Ваш идеалистический. явно продемонстрировав раздвоение суждений,

                        Каждый субъект обитает в своём собственном субъективном мире в котором он является условным центром и критерием существования всего того, что на него может воздействовать.

                        Но не факт, что нечто воздействующее на одного субъекта может также воздействовать и на другого субъекта, то есть быть также существующим и для него (например сновидение).

                        Но если что то примерно одинаково воздействует на обоих субъектов, то оно уже будет существовать для них обоих, а значит быть не только субъективно существующим, но и относительно объективно существующим.

                        Вот в этом суть субъективного идеализма.

                        Материалисты же полагают, что если нечто воздействует примерно одинаково на всех людей (материя), то оно также примерно будет воздействовать и на любых других субъектов в Мире. То есть они считают материальное абсолютно объективным.

                        И вот с этим ни я, ни Беркли, ни Юм, ни Кант, ни Шопенгауэр не согласны.

                        Мы считаем, что материальное также субъективно и существут как таковое только относительно некоторых субъектов, например людей.


                      1. Tiriet
                        23.04.2024 19:00

                        из Вашего (и из моего) определения "существует" следует, условно, "связность мира". Не может быть двух субьектов Алисы и Боба, таких, что для Алисы Цири существует, а для Боба- Цири не существует. Потому что Цири может толкнуть Алису в Боба и Боб почувствует воздействие Цири, и потому если Цири существует для Алисы, то она автоматически существует и для Боба. Между ними есть косвенное воздействие. Примерно как для меня- наличие электронов. Ни я на них, ни они на меня подействовать не можем, но вот следы воздействия электронов на мои приборы я вижу, и знаю, что где-то там электроны эти есть. А если Цири не существует для Боба- то и никаких вообще возможностей воздействовать на Боба у нее нет, и через Алису тоже! а значит и Алиса на Боба воздействовать не может, а значит, и Алисы для Боба не существует, а значит, миры Алисы и Боба- не пересекаются вообще никак и друг для друга не существуют полностью. Они, получается, несвязные "топологически".

                        Про "материалисты полагают"- акстись! мы ж спим и видим- как бы нам найти такую ситуацию, когда нечто, воздействующее на всех одинаково- на кого-то вдруг подействует иначе! это ж нобелевка прямо сразу! найди ты пример, когда импульс на что-то действует не так, как в учебнике физики- и прям сразу мировая слава, почет и уважение, передовицы и почетное звание профессора в любом ВУЗе мира! Только мы чет до сих пор не можем ничего такого найти. И поэтому вполне допускаем возможность, но считаем, что пока наша физическая реальность именно такая- объективная для всех одинаково. И полагаем, что это прям вообще так для всех без исключения! А Вы утверждаете, что где-то кто-то есть такой (уже есть! сейчас)- для которого наша физика не работает. при этом у нас есть прогностическая сила теорий, а у Вас- красота философских построений.

                        Что характерно- силикатный кирпич до сих пор существует для всех без исключения (ну, в пределах светового конуса этого силикатного кирпича- видите- мы даже знаем пределы существования чего-либо!), и для меня, и для Вас. А вот Кант- уже не существует. Хотя, когда существовал Кант- силикатный кирпич существовал и для него. Должен ли я на основании этих наблюдений решить, что силикатный кирпич- причина существования и Канта, и меня?


                      1. AleksandrTallinn Автор
                        23.04.2024 19:00

                        Про "материалисты полагают"- акстись! мы ж спим и видим- как бы нам найти такую ситуацию, когда нечто, воздействующее на всех одинаково- на кого-то вдруг подействует иначе! это ж нобелевка прямо сразу!

                        Можете уже на нобелевку подавать, так как к примеру радуга может воздействовать на человека, вызывая в нём некие ощущения, а вот на большинство животных, не обладающих цветовым зрением, она никак воздействовать не может. То есть для них она не существует.

                        Один из комментаторов к этой публикации всё это хорошо описывает https://habr.com/ru/users/Akabolka/comments/


                      1. Tiriet
                        23.04.2024 19:00

                        ну и гамма-кванты, атомные ядра и радиоволны тоже не существуют, и ультразвук и ультрафиолет. А с радугой- Вы не правы. Она для животных с монохроматическим зрением тоже существует. просто она для них не такая цветная, как для нас.

                        И комментарий, Вами приведенный, очень ярко описывает, только не это все, а сам подход. Прям второе предложение " объективной реальности не существует по определению" если у нас есть определение "объективной реальности", согласно которому "она не существует"- так я ж согласен! определение только приведите, чтоб быть уверенным, что мы с одним и тем же определением живем :-) Я с этого и начал с Вами дискуссию- с вопроса- а где определения? Где, .... его, определения!?


                      1. AleksandrTallinn Автор
                        23.04.2024 19:00

                        А с радугой- Вы не правы. Она для животных с монохроматическим зрением тоже существует. просто она для них не такая цветная, как для нас

                        Это что то новенькое -"нецветная радуга" :)


                  1. victor_1212
                    23.04.2024 19:00

                    Согласитесь, что все наши слова, определения, это наш человеческий способ общения между собой по некоей условной договорённости.

                    договоренность может быть "встроенной", т.е. просто мозг и сознание устроены похожим образом, это касается также общения с животными, и общения животных разных видов между собой, которые вполне могут общаться и дружить,

                    Кстати это одна из важнейших математических проблем - почему мир вообще можно описать математически и иногда даже слишком просто

                    одно из возможных объяснений - структура нашего сознания/подсознания такова, что способна развиваться создавая новые образы и абстракции, подобно новым api дающим доступ к слоям подсознания, и позволяющим применять методы интуиции в новых областях знаний


                1. kryvichh
                  23.04.2024 19:00

                  Если мощный компьютер перегорел от напряжения при расчёте очень важной задачи, то мы должны рассматривать результаты его работы ошибочными? Так и с Ницше. И с многими другими людьми, которые "сломали" свой "компьютер"-мозг в результате непомерных нагрузок, но успели оставить нам свои труды (философские, научные, музыку, стихи, картины).


                  1. Tiriet
                    23.04.2024 19:00
                    +1

                    Вот Вы на аналогиях там мило подставляетесь. Если "мощный компьютер" сгорел- то мы действительно должны результаты его работы рассматривать как ошибочные. потому что он работал с перегревом и значит в расчетах были артефакты. именно по этой причине расчетчики на игровых GPU все расчеты прогоняют дважды и побитово сравнивают результаты, и только если совпадение побитовое точное- эти результаты принимаются за достоверные. потому что в горячих ГПУ при проектировании заложено, что 1 артефакт на миллиард треугольников- это допустимо. а для расчетов- не допустимо.

                    Второе. Вы предполагаете, что у мозга может быть какая-то "непомерная нагрузка". Данные физиологии с Вами не согласны- мозг всегда работает на более чем 90%, и никакой "непомерной" нагрузки в принципе не допускает. и уж совсем неправдоподобно, что непомерная нагрузка была у Ницше, кроме того, психиатрия (насколько я с ней знаком) утверждает, что внезапных сходов с ума не бывает- люди сходят с ума сильно заранее до появления явных клинических симптомов. В случае с Нишце вся его философия- это медленно развивающееся душевное заболевание :-). не "он сначала нафилософил, а потом сошел с ума", а он сошел с ума, нафилософил в процессе прогрессирующего заболевания и под конец уже на фоне яркого эмоционального переживания сошел окончательно и явно. И другие люди- они не сошли с ума в результате "непомерных трудов" и оставили труды, а они сначала сошли с ума, и именно из-за этого нам их труды оказались настолько гениальнымы- потому что мы, нормальные, не можем отличить в какую сторону видим отклонение от нормы, если это отклонение не очень большое. оно нас просто привлекает своей отклоненностью. которую мы зачастую воспринимаем как гениальность.


                    1. kryvichh
                      23.04.2024 19:00

                      Также как результаты компьютерных вычислений можно проверить, так и результаты умственного труда изучаются, и выносятся суждения экспертами. Уж что, а труды Ницше исследовали вдоль и поперек. И мемов он наплодил достаточно. Раз многие из них живут до сих пор, значит, соответствуют картине мира других людей.

                      Мозг вполне может работать со сверхнагрузкой, если человек устал, болен, или находится под действием "веществ". Кто-то научился с этим жить и восстанавливаться, кто-то заболел или потерял рассудок. Вон программисты на раз "выгорают".


                      1. Tiriet
                        23.04.2024 19:00
                        +2

                        труды Ницше нельзя проверить также как результаты расчетов. Они (труды Ницше) носят принципиально другой характер, нежели труды математиков. Мем про "превед медвед" живет в узких кругах до сих пор. Разве из этого следует, что он соответствует картине мира людей?

                        Про "сверхнагрузку"- повторяю- те данные по физиологии, которые видел я- вообще отрицают возможность работы мозга с какой-то там регулируемой нагрузкой. Он всегда работает на полную мощность, просто переключается между разными задачами. И кстати- работа сознания- далеко не самая ресурсоемкая работа мозга. реально ресурсоемкое- это настройка ЖКТ на переваривание котлетки. или управление движением тушки от кроватки до работки. И выгорание- опять же не имеет никакого отношения к нагрузке на мозг- программисты не нагружают его как-то там особенно по сравнению с, например, грузчиками или комбайнерами. Выгорают и те профессии, которые вроде как вообще интеллект не используют. Просто они не пишут об этом в интернетах, как программисты. И даже выгоревшие- не сходят с ума. Психиатрия- это физиологические нарушения в работе мозга- на уровне биохимии. А выгорание- это психологическое- это принципиально разные вещи.


                      1. kryvichh
                        23.04.2024 19:00

                        Хроническое выгорание может вести к психическим расстройствам. "Принципиально разные" - ну-ну. Спор ради спора...


                      1. Tiriet
                        23.04.2024 19:00

                        это уже узко-специальный вопрос. но из того опыта общения с психиатрами и психотерапевтами, который есть у меня- психические расстройства основаны на нарушениях физиологии мозга (потому их и лечат лекарствами! корректируют активность нейронов принудительно), а то, что проявления этих расстройств обостряются при "хроническом выгорании" (это кстати какой-то бытовой термин, в МКБ такого нету)- ну так да, могут- но для этого надо, чтобы расстройство уже было. Шизофрения передается генетически, а выгорание- нет. Можно уронить плохо-прикрученный столб легким касанием, но только если он уже плохо прикручен, а нормально прикрученный столб не падает даже от шквального ветра.

                        впрочем, должен отметить, что я так же читал и технологии создания шизофрении в здоровом мозгу- что несколько противоречит теории о врожденных физиологических причинах психиатрических заболеваний, но там технологии такие, что сами по себе очень сильно сдвигают физиологию (гормоны там шпарят на сильно повышенных уровнях). Однако, Ницше под эти технологии точно не попадал.


                    1. victor_1212
                      23.04.2024 19:00

                      они сначала сошли с ума, и именно из-за этого нам их труды оказались настолько гениальнымы- потому что мы, нормальные, не можем отличить в какую сторону видим отклонение от нормы, если это отклонение не очень большое. оно нас просто привлекает своей отклоненностью

                      это объяснение предполагает четкое определение "нормальности", которое подобно определению "сферических коней в вакууме" является абстракцией удобной для описания большинства, тогда как в реальности при определенных обстоятельствах нервная система любого человека может не выдержать, разница в чувствительности, и если у человека что называется "бычьи нервы", это не делает его более нормальным, все зависит от обстоятельств и природных способностей, грубо говоря чем более человек понимает, тем менее счастлив от этого, и признаки нормальности исчезают, больше не как все :)


              1. alysnix
                23.04.2024 19:00
                +5

                не стоит к познанию мира привлекать философов. обычно это люди безграничные в полете фантазии, и крайне ограниченные в реальном знании явлений материального мира.


                1. AleksandrTallinn Автор
                  23.04.2024 19:00

                  не стоит к познанию мира привлекать философов

                  Вы просто не обращайте на них внимания, пусть что то там себе фантазируют :)


                1. Imaginarium
                  23.04.2024 19:00

                  Привлекать к изучению вопросов мироздания философов можно и нужно, вот только где их взять? Всё обсуждаемое здесь словоблудие в статье возникло как раз из-за профанации философии, и будет ещё хуже, если нормальное гуманитарное образование не удастся восстановить. Будет всё больше идиотских, высосанных из пальца статей без минимальной логики и понятийного аппарата, вроде этой, построенной на умозрительных связях между не определенными нигде явлениями.


            1. AleksandrTallinn Автор
              23.04.2024 19:00

              Вас все субъективно и зависит от наблюдателя

              Да, я считаю (вместе с Беркли, Юмом, Кантом, Шопенгауэро, Ницше, Шпенглером и т.д.) что всё субъективно и если мы начинаем что либо описывать и о чём либо рассуждать не указывая относительно какой системы отсчёта (наблюдателя) мы это делаем, то это всё становится бессмысленным.

              Это всё равно что определять расстояние, не указывая первоначальной точки отсчёта


              1. Tiriet
                23.04.2024 19:00
                +2

                :-) с Вами интересно.

                для меня система отсчета- вполне себе конкретный термин, и его смысловое содержание не позволяет использовать его в виде "рассуждать в системе отсчета". А Вам- позволяет. потому что у Вас "система отсчета"- какая-то лично Ваша формулировочка, которая непонятно что значит. Вам-то может и понятно, что Вы там имели в виду, а окружающим- типа меня- нет. Возможно, что Беркли, Шпенглер и остальные- тоже имели такое же понятие о "системе отсчета", как и Вы- и тогда там у Вас все с ними будет консенсусно (но я уверен, что они, как и Вы- что-то свое имели в виду, поэтому консенсус там если и есть- то только мнимый).

                А вот Ваша сентенция про "расстояние"- она для меня вообще показательна! потому шта мне в физике для измерения расстояния точка отсчета не нужна! мне нужны точки, между которыми я это расстояние измеряю! и оно (расстояние) в любой системе отсчета будет одинаковое! Парампарампам. при этом "расстояние" у меня тоже будет вполне себе четко определенный термин, хоть в Ньютоновской механике, хоть в ОТО :-). разный, но четко определенный и там, и там. А я, в своих суждениях, буду уточнять- какое именно расстояние имею в виду, и все другие физики всех других рас и биологических (или небиологических) форм будут меня понимать и понимать одинаково! :-) А Вам вот надо зачем-то точку отсчета для определения расстояния, интересно, однако.


                1. AleksandrTallinn Автор
                  23.04.2024 19:00

                  и оно (расстояние) в любой системе отсчета будет одинаковое!

                  Вот на этом месте Эйнштейн заворочался в гробу, а вместе с ним Лоренц и многие остальные, которые утверждали, что "расстояние между двумя точками тела не является абсолютной величиной, а зависит от скорости движения тела относительно данной системы отсчёта."


                  1. Tiriet
                    23.04.2024 19:00

                    они же и выкрутились- они "интервал" придумали, и все снова стало понятно и объективно. поэтому я Вам и сказал, что у меня "расстояние" в ньютоновской механике и в ОТО будет по-разному определено, но четко и одинаково для всех.


                    1. AleksandrTallinn Автор
                      23.04.2024 19:00

                       у меня "расстояние" в ньютоновской механике и в ОТО будет по-разному определено, но четко и одинаково для всех.

                      Расстояние между двумя точками на поверности шара можно высчитать минимум тремя разными способами


                      1. Tiriet
                        23.04.2024 19:00
                        +1

                        определить! а после определения того, что мы в конкретном случае называем "расстоянием"- оно уже высчитывается только одним способом. потому что слова- это обозначения смыслов! и разные смыслы можно обозначать одним словом (особенно если смыслы близкие). Поэтому-то физики и цепляются к формулировкам- потому что четкие формулировки позволяют локализовать смыслы, и быть уверенным, что мы говорим про одни и те же смыслы, а не каждый про свое. А Вы мне сейчас пытаетесь доказать, что одно слово может иметь разный смысл- ну да. может. поэтому я Вам и начал задавать вопросы- какие конкретно смыслы Вы закладывали в свои слова. и чем дальше- тем больше понимаю, что у нас с Вами принципиально разный способ мышления. Вы, условно, лефевровский лирик, а я- условно лефевровский физики. Вы про ассоциативные связи между образами, а я- про логические между смыслами.


                      1. kauri_39
                        23.04.2024 19:00
                        +1

                        Не можете ли вы пояснить, почему плотность тёмной энергии вакуума выражают в килограммах на кубометр, хотя она, в отличие от гравитирующей материи/энергии, проявляет себя противоположным образом - расширяет пространство-время? И её включают в состав общей массы Вселенной, приписывая ей около 68 % массы. Как будто это горячий газ, расталкивающий материю...


                      1. AleksandrTallinn Автор
                        23.04.2024 19:00

                        Не можете ли вы пояснить, почему плотность тёмной энергии вакуума выражают в килограммах на кубометр, хотя она, в отличие от гравитирующей материи/энергии, проявляет себя противоположным образом - расширяет пространство-время?

                        Известно, что "тёмная энергия" была введена в космологию как один из вариантов объяснения процесса расширения Вселенной. Она участвует в гравитационном взаимодействии (и только), а значит должна обладать массой


                      1. Tiriet
                        23.04.2024 19:00
                        +1

                        потому что у темной энергии уравнение состояния такое- при положительной плотности она дает отрицательное давление. мы не видим примеров такой ерунды в нашей бытовой жизни, но мы видим расширение Вселенной, которое вполне сносно объясняется и понимается, если мы допускаем, что в ней тоненько-тоненько но по всему объему размазана именно такая субстанция, которая имеет положительную энергию-массу, но отрицательное давление. а дальше нам пофиг на то, понятно это кому-то в интернетах, или нет- мы просто построили модель, расчеты по которой показывают похожее на то, что показывают телескопы, а почему модель такая- это уже к теор-физикам, про такое на хабре в комментах спрашивать бесполезно.


              1. hurtavy
                23.04.2024 19:00

                Дайте, пожалуйста ваши определения объективного и субъективного. Ну или хотя бы больше примеров отличия "субъективных" законов физики от "объективных"


                1. AleksandrTallinn Автор
                  23.04.2024 19:00

                  Дайте, пожалуйста ваши определения объективного и субъективного. Ну или хотя бы больше примеров отличия "субъективных" законов физики от "объективных"

                  Вокруг нас масса примеров относительной объективности. Многие люди видят красный свет светофора, а дальтоники (и многие животные) нет. Значит красный цвет лишь относительно объективен.

                  Что касается законов физики, то они всегда объективны (личных законов физики не бывает), но тоже существуют относительно той или иной субъективной системой отсчёта (например связанной с общечеловеческим восприятием мира)

                  Хотя можно сказать, что когда в голове Ньютона родился закон притяжения массивных тел, то он не был объективным, а исключительно субъективным, личным взглядом Ньютона на мир. А все другие люди для объяснения мира использовали разные другие субъективные закомерности.

                  Потом когда с идеей Ньютона согласились другие учёные, то закон притяжения стал относительно объективным.

                  Но Эйнштейн доказал, что законы притяжения Ньютона это некая приблизительная условность, которую можно использовать в некоторых условных системах отсчёта не требующих большой точности.


            1. hurtavy
              23.04.2024 19:00

              Так статья и есть про философию, а не про науку. Вот сразу же видно по первому абзацу. Науке, и тем более физике, безразлична граничность познания, пока мы в эти границы не упрёмся. А мы не упёрлись, мы даже не представляем, где эти границы.


              1. gitorion
                23.04.2024 19:00

                Уперлись! Граница - энергия связи кварков для преодоления которой нужен коллайдер размером с Юпитер. Экспериментальных данных уже нет лет 50, поэтому приходится фантазировать и прорабатывать теорию струн, которая по сути это одна из фантазий, как бы мог быть устроен мир глубже кварков. И даже если удастся расщепить кварк, там следующий слой, наверняка еще с более изощренной структурой. А самая непреодолимая граница в пределе движения вглубь, если задаться вопросом - а сколько там вглубь еще слоев, и на чем базируется последний? Или количество слоев бесконечно?


                1. hurtavy
                  23.04.2024 19:00

                  а причём тут отсутствие приборов к принципиальной непознаваемости?


                  1. gitorion
                    23.04.2024 19:00

                    А как познавать без приборов?


                    1. hurtavy
                      23.04.2024 19:00

                      Почему без? То, что такого прибора нет сегодня, не значит, что мы его не создадим завтра или через 10, через 100 лет... Тем более, что мы даже знаем, что для этого надо.


            1. saag
              23.04.2024 19:00

              Мир - (предположительно) область пространства, в которой образовался гравитационный пузырь после выброса туда огромного количества после БВ;

              Пространство - место куда поместился(-лись) гравитационный пузырь(-и), в общем многомерный носитель.

              Вселенная - гравитационный пузырь, в котором материя остывая образовала газовые скопления, звезды, галактики;

              Солнечная система - полигон для создания биоразнообразия, планета пригодная для отработки биологических экспериментов была создана с 4-й попытки - Фаэтон - Марс - Венера - Земля, стабильное биоразнообразие удалось создать со второй попытки, после первой пришлось планету стерилизовать методом удара астероида.

              Консоль админа до сих пор не найдена.


              1. Tiriet
                23.04.2024 19:00
                +1

                у Вас пространство определено через пузырь, пузырь - через пространство, а Вселенная- через пузырь, вероятно, Вам следует погуглить про сепульки для ознакомления с имеющимися литературными данными по парадоксам подобного рода.


                1. saag
                  23.04.2024 19:00

                  может немного запутано, но где вы видели ясность в этом вопросе?


            1. AleksandrTallinn Автор
              23.04.2024 19:00

              опять вопрос- что значит "объективное"? у меня, как у физика, объективное- это которое проверяемое другими исследователями и после проверки сохраняющее формулировки

              В моей публикации постоянно отмечается, что абсолютной объективности не существует, но относительной объективности, таковой для нескольких общающихся между собой субъектов сколько угодно.

              Если нечто открытое одним экспериментатором подтверждается и другими экспериментаторами то это опять же относительная объективность.

              То есть я предполагаю, что в других Вселенных (а возможно даже и нашей тёмной материи) существуют субъекты с которыми у нас никогда не будет общей объективности. Они воспринимают мир совершенно иначе.

              Любые наши определения "случайность", "масса" и даже "объективность" - субъективны, то есть ими можно пользоваться только в той или иной условной системе отсчёта (каковой, в частности, является "человечество")


            1. Tarkal
              23.04.2024 19:00
              +1

              Я, как физик, читаю Вас и задаюсь вопросом - а Вселенная это что? Мир- это что?

              Вы позиционируете себя как "физика", однако задаваемые вами вопросы носят характер сугубо философский. В советском ВУЗе учились? "Диамат" проходили? Первую лекцию не прокурили на лестнице?

              "Материя (сиречь мир, вселенная) есть объективная реальность, данная нам в наших ощущениях" - помните? У меня сложилось впечатление, что автор понимает мир совершенно адекватно. В чем проблема?

              Вы мне про какую-то неизвестную субстанцию рассказываете...

              Не передергивайте. Буквосочетание "субстанция" в авторском тексте как таковое отсутствует.

              Вы рассуждаете про "абсолютное знание". А я опять задаюсь вопросом- "абсолютное"- это что за характеристика?

              Возможно, автор просто не считает нужным подробно разжевывать общеизвестные вещи? Во-всяком случае, для ученых. Мне лично различия между знанием абсолютным (аксиоматическим) и относительным (постулатом) вполне доходчиво растолковал школьный учитель геометрии на первом же уроке. В порядке ликбеза:

              Знание абсолютная или аксиома – утверждение безусловно верное, не нуждающееся в доказательствах в силу своей самоочевидности, заведомо исключающее альтернативу и в принципе не допускающее возможности опровержения ни логически, ни каким бы то ни было опытом.

              Примером могут служить аксиомы математики: "В числе трех точек, лежащих на незамкнутой линии всегда есть одна, и только одна, расположенная между двух других".

              Если есть сомнения, можете попробовать расположить их иначе - хотя бы с тремя горошинами. Сообщите, с какой попытки это у вас получилось - вариантов не так уж много.

              Вы рассуждаете про "объективное знание". и опять вопрос - что значит "объективное"? У меня, как у физика, объективное - это которое проверяемое другими исследователями и после проверки сохраняющее формулировки.

              У меня тоже. Полагаю, как и у автора. Объективное знание о чем бы то ни было всегда относительно - сколько бы вы не проверяли тот или иной постулат, возможность, что при каких-то исходных параметрах результат опыта будет отличен от предсказываемого теорией, АБСОЛЮТНО не исключена, и автор совершенно прав: "Абсолютно объективное знание о Мире невозможно, это просто миф".

              И все Ваши выводы для меня выглядят как бессмысленные. то есть, ассоциативные связи между терминами в этих выводах я вижу, а вот логические связи - отсутствуют, потому что термины в этих выводах не имеют определений...

              Не требуйте от автора слишком многого. Как я понял, текст представляет собой популярное изложение взглядов индивида, и на статус законченной научной теории не претендует. Если некие ассоциативные связи таки уловили и конкретных принципиальных возражений по существу сказанного не имеете, что мешает вам, как профессионалу, поразмышлять и над приемлемой терминологией?

              ... прочитал с интересом. как пример другого в корне, не связного и поверхностного мышления.

              Я тоже. Читается легко, язык прост и понятен, текст содержит немало новых (для меня, во всяком случае) оригинальных мыслей. Подумать есть над чем.


          1. V_Scalar
            23.04.2024 19:00

             В нашей Вселенной случайным образом сложились условия для возникновения субъектности. Предположительно большинство Вселенных в Multiverse мертво, а в те Вселенные где другие какие-то субъекты возможно есть нам никогда не попасть

            я представляю так: вакуум существовал всегда и до большого взрыва, имелт строго определённые свойства — все слабые заряды имели левую киральность (слабый изоспинТ3, а правые несут слабый гиперзаряд взаимодействуют с нейтральными токами. То есть структура вещества была заранее предопределенна. Достаточно было создать холм или запас энергии от большого взрыва с последующим расширением и охлаждением, и вещество стало выпадать в осадок из этой плазмы


        1. debagger
          23.04.2024 19:00
          +1

          Такая лотерея, где выигравший получает сознание, чтобы осознать что он выиграл ))


          1. Ergistael
            23.04.2024 19:00

            Уловка-22 ;)


  1. Imaginarium
    23.04.2024 19:00
    +7

    Пришло время разрушить ещё одно неверное представление, о том, что человек это существо, способное генерировать абсолютно объективное знание о Мире с помощью своего сознания.

    Поздравляю, Вы только что изобрели непознаваемого Бога (Он Вам необходим как аспект бытия в Вашем построении), но как-то не до конца. Познаваемость мира является не аксиомой, а гипотезой, и Вы, пытаясь опровергнуть её, предлагаете новую, которая тоже пытается объяснить природу мира, причём сразу финальным образом, до конца. То есть, Вы меняете шило на мыло. Лучший способ попытаться опровергнуть предполагаемую познаваемость мира -- не писать об этом на Хабре статьи просто не выдвигать об этом никаких контргипотез.


    1. AleksandrTallinn Автор
      23.04.2024 19:00
      +1

      Познаваемость мира является не аксиомой, а гипотезой, и Вы, пытаясь опровергнуть её, предлагаете новую

      Я не отрицаю познаваемость мира, а только пишу о том, что любое познание мира субъективно, условно, относительно и его основной целью является продолжение существования познающего субъекта


      1. Tiriet
        23.04.2024 19:00

        то есть, из Вашего этого утверждения следует, что вообще цель может быть только одна- продолжение существования?

        Я почему спрашиваю- потому что лично я искренне считаю, что цели могут быть разные, их больше, чем одно продолжение существования (например- удовольствие от существования или наоборот- прекращение существования!) и что множество связей "действие"-"цель" не является только "многие к одному", а является связью "многие ко многим" (в терминологии реляционных баз данных :-)). то есть, возможно "познание мира с целью прекращения существования", возможен "отказ от познания с целью продолжения существования", возможно "познание с целью удовольствия" или "познание с целью снижения страданий" или "познание от нечего делать", вообще без цели, но со стимулом :-)

        Вам критически не хватает четкости формулировок :-)


        1. V_Scalar
          23.04.2024 19:00

          представляю так: вакуум существовал всегда и до большого взрыва, имел строго определённые свойства — все феромоны имели левую киральность (слабый изоспинТ3±½ взаимодействует W , а правокиральные феромоны взаимодействуют только с нейтральными токами несущими слабый гиперзаряд Y взаимодействует с Z. То есть структура вещества была заранее предопределенна. Достаточно было создать холм или запас энергии от большого взрыва с последующим расширением и охлаждением, это вызвало бы поток энергии вниз по градиенту холма. Ну а далее по Ильи Пригожину, самоорганизация в неравновесных системах , вещество стало выпадать в осадок из этой плазмы


          1. Tarkal
            23.04.2024 19:00

            все феромоны имели левую киральность (слабый изоспинТ3±½ взаимодействует W, а правокиральные феромоны взаимодействуют только с нейтральными токами несущими слабый гиперзаряд Y взаимодействует с Z.

            Скорее всего, Вы правы, однако все же рецессивная аллель влияет на фенотип только тогда, когда генотип гомозиготен.


        1. AleksandrTallinn Автор
          23.04.2024 19:00

          то есть, из Вашего этого утверждения следует, что вообще цель может быть только одна- продолжение существования?

          Продолжение существования субъекта его главная цель - потому что не будь самого субъекта не было бы смысла говорить о каких то других связанных с ними целях.

          Но субъект лишь условное единое, цельное существо. На самом деле он состоит из условных частей, каждая из которых может иметь и свои собственные "цели".

          К примеру "мемы" это такие части субъекта, которые могут привести к его самоуничтожению. В частности мем "суицида" распрострающийся среди людей часто приводит к гибели самого субъекта- человека


      1. hurtavy
        23.04.2024 19:00

        Точно также можно предположить, что целью познания является уничтожение субъекта.


        1. Tarkal
          23.04.2024 19:00
          +1

          ... можно предположить, что целью познания является уничтожение субъекта.

          Можно. Цивилизации, в которых доминирует такая "цель" просто самоуничтожаются. Естественный отбор.


      1. Imaginarium
        23.04.2024 19:00

        Когда Вы осилите какую-нибудь брошюрку про такое философское направление как солипсизм (к нему, в пределе, сводятся всё Ваши, ээээ, рассуждения), а затем худо-бедно хотя бы в какой-нибудь Википедии прочтёте про его критику, Вам самому станет стыдно за эту "статью".


        1. Tarkal
          23.04.2024 19:00
          +2

          Когда Вы осилите какую-нибудь брошюрку про такое философское направление как солипсизм...

          Тоже философия, не хуже прочих. Если имеете за плечами советское ВО, преподаватель философии не открыл вам на первой же лекции "страшную тайну", что из всех философских систем понятию "абсолютная истина" отвечает только субъективный идеализм - самое, казалось бы, нелепое допущение?

          В.И.Ленин, "Материализм и эмпириокритицизм": Представим себе последовательного идеалиста, который стоит, положим, на той точке зрения, что весь мир есть мое ощущение или мое представление и т. д... Идеалист и не подумает отрицать того, что мир есть движение, именно: движение моих мыслей, представлений, ощущений. Вопрос о том, что движется, идеалист отвергнет и сочтет нелепым: происходит смена моих ощущений, исчезают и появляются представления, и только. Вне меня ничего нет. "Движется" - и баста. Более "экономного" мышления нельзя себе представить. И никакими доказательствами, силлогизмами, определениями нельзя опровергнуть солипсиста, если он последовательно проводит свой взгляд.


          1. Imaginarium
            23.04.2024 19:00

            Я не оперирую понятием "абсолютная истина", зато Вы явно подходите к философии как гигантскому супермаркету, где можно выбрать получше. Это говорит о том, что Вы явно с субъективистских позиций оцениваете субъективизм (закономерно положительно). Попробуйте дочитать процитированную книжку.


  1. Radisto
    23.04.2024 19:00
    +6

    Часть с антропологией содержит такое количество фактических ошибок и неверных датировок, что лучше ее вообще удалить. Ну или полностью переписать, проконсультировавшись с археологами или хотя бы найти достоверную информацию. Сейчас там половина уровня Рен-ТВ


    1. Dolios
      23.04.2024 19:00
      +2

      Да там вся статья такая, к сожалению.


    1. AleksandrTallinn Автор
      23.04.2024 19:00
      +1

      Часть с антропологией содержит такое количество фактических ошибок и неверных датировок

      Пожалуйста, приведите хотя бы одну фактическую ошибку в антропологии в этой публикации


  1. cbdk00
    23.04.2024 19:00
    +3

    Доставляющая философская интоксикация!


  1. alysnix
    23.04.2024 19:00

    бесконечность - это математическая абстракция. в физике бесконечностей быть не может. потому не может быть "бесконечного количества вселенных".

    математическое рассуждение выводящее бесконечность, навроде - ну раз есть число N, то почему не быть числу N + 1, для физики и материального мира неверно в принципе. N+1 не может быть потому, что для этого нужна энергия. а ее больше нет.


    1. AleksandrTallinn Автор
      23.04.2024 19:00

      в физике бесконечностей быть не может. потому не может быть "бесконечного количества вселенных".

      Согласен, что в нашем человеческом восприятии (а наука физика часть нашего восприятия) всё более-менее конечно, потому что бесконечность мы просто никак не можем воспринять (и даже сложно себе представить).

      Бесконечость для нас это некая абстракция - модель для рассуждений и гипотез в том случае, когда конечность вообразить и смоделировать бывает ещё сложнее.

      Допустим вы как то можете себе представить конечность нашей Вселенной. Ну а что дальше за ней, в остальном Мире?

      Как вариант ответа - бесконечное количество других Вселенных, потому что если их конечное количество, то возникает тот же вопрос - а что дальше за ними?

      Ограниченность Мира представить гораздо труднее, чем его безграничность


      1. Tarkal
        23.04.2024 19:00
        +1

        Ограниченность Мира представить гораздо труднее, чем его безграничность

        И то и другое одинаково ужасно.


    1. Tarkal
      23.04.2024 19:00
      +1

      бесконечность - это математическая абстракция. в физике бесконечностей быть не может. потому не может быть "бесконечного количества вселенных".

      Такая же абстракция, как и понятие бесконечно малой (безразмерной) точки. Напомню, что у Евклида точка это нечто отнюдь не бесконечно малое, но «… то, что не имеет частей», т.е. ЭЛЕМЕНТАРНО малое, однако вполне себе реальное.

      Тем не менее, это не мешает физикам оперировать представлением о системах отсчета, началом которых служат координаты X=0, Y=0, Z=0, Т=0. И, как следствие - приводит к абсурдному выводу о возникновении Вселенной "из ничего".

      N+1 не может быть потому, что для этого нужна энергия. а ее больше нет.

      С чего вы взяли, что нет? Коран: "Каждое мгновение Аллах уничтожает мир (сиречь, Вселенную), и каждое мгновение вновь создает его" - из предыдущего состояния. И так бесконечное количество раз. В чем проблема?


      1. Imaginarium
        23.04.2024 19:00

        Проблема в привлечении божественной сущности для описания движения и происхождения материи, что неизмеримо большее допущение, чем введение любых актуальных бесконечностей в любые теории.


  1. ababich
    23.04.2024 19:00
    +1

    если на Хабре есть хаб философия, то вам следует там публиковать такие вещи... тем более если вы любитель не ограничивать себе размерами публикации... увы, плохо, когда количество не переходит в качество ...

    а если серьезно, то достаточно прочитать начало статьи и резюме ... для прочтения этого достаточно :) ... да и при написании можно было этим ограничиться :))

    Итак :

    Современная наука, и физика в частности, имеют в своей основе одну, как я полагаю, ложную аксиому. И если её поправить, то всё научное мировоззрение значительно изменится и многое станет гораздо более ясным. Это аксиома о том, что человеческие способности познания Мира безграничны и рано или поздно, при более совершенных приборах и способах исследования, мы познаем всё и до конца. А наше знание о Мире абсолютно объективно.

    нет, конечно :)

    нет такой "аксиомы" ... если говорить о науке ...

    такая "аксиома" есть в голове у взрослых людей, которые так и не переросли ментальность подростка ... поэтому такие люди уверены, что познание бесконечно, что технический прогресс- тоже ... как следствие - они рассказывают о межзведных перелетах и о колонизации чего-то там где-то там :)))

    а на самом деле процесс познания ограничен доступными нам энергиями ... вот и все ... все очень просто ... а наши технологии в основном ограничены законами физики ... ну и еще одно ограничение : мы о мире в широком смысле слова не все можем знать много и исчерпывающе по простой причине : мы что-то (не все) знаем о нашей Вселенной (по сути не зная, что она такое ... ибо для этого нам бы надо "выйти за ее пределы" (это образное выражение) ...а это невозможно) ... ну и еще наши возможности сильно ограничены тем, что мы хрупкие белковые существа и обречены быть тут ...в пределах тропосферы... все что далее не пригодно для нашего существования... мы слишком хрупкие ... родились и эволюционировали именно тут ... в этом райском уголке ... если говорить об условиях в тропосфере

    Пожалуй ваша ошибка в том, что вы рассказываете , что наше знание субъективно ... если я вас правильно понял. На самом деле оно объективно, просто оно в принципе ограничено доступными нам энергиями, доступными нам технологиями, ограничено законами физики, ограничено тем, что у нас один экземпляр Вселенной.

    Но "красный цвет" - это не такая уж субъективная вещь.

    Так как ЭМ волна - вещь объективная. И даже ее длина- вещь объективная.

    А все остальное - это предмет наших договоренностей. Измерять длину ЭМ волны в тех единицах, что мы договорились. И называть это "красный" (так договорились).

    И коротко о "резюме".

    Да, Волга действительно впадает в Каспийское море :)

    Действительно возникла Вселенная... потом возникла жизнь.

    А дальше - некорректно о субъективности нашего знания. Это я уже прокомментировал.

    Оно объективно. Но ограничено (по вполне объективным причинам) :))))


    1. AleksandrTallinn Автор
      23.04.2024 19:00

      на Хабре есть хаб философия

      На Хабре нет такого хаба

      Пожалуй ваша ошибка в том, что вы рассказываете , что наше знание субъективно

      нет, это не моя ошибка, я в этом убеждён вместе со многими авторитетными философами, котрых я цитирую.

      А дальше - некорректно о субъективности нашего знания. Это я уже прокомментировал.

      Оно объективно.

      Наше знание относительно объективно и поэтому применимо для ограниченного количества субъектов


      1. ababich
        23.04.2024 19:00

        Наше знание относительно объективно и поэтому применимо для ограниченного количества субъектов

        перед тем, как говорить о чем -то следует определиться с терминами и понятиями , предлагаю сделать это

        что такое "Наше знание" ? Это то, что называется "наука" ?

        Или вы предлагаете заменить науку чем-то другим?

        Чем?

        Может ощущениями человека?

        Наше знание относительно объективно

        что значит "относительно объективно" ?

        это ваш собственный термин (придуманный вами) ? Или вы его почерпнули где-то?

        Приведите примеры пожалуйста.

        человек это существо, способное генерировать абсолютно объективное знание о Мире с помощью своего сознания.

        если говорить о науке, то научное знание о мире основано на показаниях приборов, а не на сознании

        Вокруг нас масса примеров относительной объективности. Многие люди видят красный свет светофора, а дальтоники (и многие животные) нет. Значит красный цвет лишь относительно объективен.

        странно, что вы взяли дальтоника...

        можно же взять слепого :)))))

        он не видит свет, то есть не воспринимает ЭМ волны

        в диапазоне длин от 380 нм до 780 нм

        то есть ЭМ волны в этом диапазоне "относительно объективны" :))))))))

        Когда Вы осилите какую-нибудь брошюрку про такое философское направление как солипсизм (к нему, в пределе, сводятся всё Ваши, ээээ, рассуждения), а затем худо-бедно хотя бы в какой-нибудь Википедии прочтёте про его критику, Вам самому станет стыдно за эту "статью".

        вам советовали про соллипсизм почитать?

        вы вняли совету?

        вам вообще дают хорошие бесплатные советы....увы... похоже это бесполезно

        Согласен, что в нашем человеческом восприятии (а наука физика часть нашего восприятия) 

        наука физика к восприятию отношения не имеет, она основана на показаниях приборов

        в широком смысле слова

        Я не отрицаю познаваемость мира, а только пишу о том, что любое познание мира субъективно, условно, относительно и его основной целью является продолжение существования познающего субъекта

        если мы о науке, то познание мира основано на показаниях приборов, то есть объективно, цели у него нет как таковой (это любопытство (что характерно для человека) + чисто утилитарные моменты) ....а что такое "условно, относительно" - это только вам известно.... и познание ограничено (доступными нам энергиями и тем, что у нас только один (этот) экземпляр Вселенной ... его мы только и можем изучать)

        В моей публикации постоянно отмечается, что абсолютной объективности не существует, но относительной объективности, таковой для нескольких общающихся между собой субъектов сколько угодно.

        Если нечто открытое одним экспериментатором подтверждается и другими экспериментаторами то это опять же относительная объективность.

        при чем тут общение? :)))

        открытие подтверждается тем, что в другом месте и в другое время (например) приборы зафиксировали то же самое :))) ...


        1. AleksandrTallinn Автор
          23.04.2024 19:00

          что такое "Наше знание" ? Это то, что называется "наука" ?

          Пусть будет так, хотя термин "знание" несколько шире термина "наука"

          что значит "относительно объективно" ?

          это ваш собственный термин (придуманный вами) ? Или вы его почерпнули где-то?

          Этот термин я использую давно, уж даже и не помню как он пришёл ко мне, может ещё кто то использует не знаю точно.

          Он означает что "нечто" воспринимается "примерно одинаково" различными субъектами.

          Например сновидение воспринимается каждым человеком сугубо индивидуально и поэтому не является "относительно объективным".

          Радуга воспринимается примерно одинаково всеми людьми поэтому является "относительно объективной" для них, но тоже не абсолютно объективной, так как не воспринимается некоторыми животными.

          На мой взгляд не существует ничего абсолютно объективного.

          Допустим мы стали общаться с какими то инопланетянами. И тогда мы выясняли бы с ними что из нашего мира они воспринимают, а что нет. И мы вместе с ними формировали бы нашу совместную "относительно объективную реальность"

          то есть ЭМ волны в этом диапазоне "относительно объективны"

          ЭЛМ волны это возмущение ЭЛМ полей которые мы определяем по воздействию на что либо более менее хорошо нами воспринимаемое. И это воздействие различными субъектами может быть воспринято по разному или не воспринято вообще. Поэтому оно всегда относительно объективно.

          Летучая мышь для ориентации в пространстве использует звуковые волны, мы используем ЭЛМ волны. Мир летучей мыши отличается от нашего мира довольно радикально.

          Вы можете долго спорить с летучей мышью о существовании или не существовании чего либо.

          И даже ChatGPT не считает ЭЛМ волны материальными (в своей публикации я даю ссылку на это) и обозначает их как "субматериальные", то есть не совсем материальные.

          По всей видимости тёмная материя никак не контактирует с ЭЛМ волнами, это ещё один аргумент за то что ЭЛМ волны не абсолютно объективны

          вам советовали про солипсизм почитать?

          вы вняли совету?

          вам вообще дают хорошие бесплатные советы....увы... похоже это бесполезно

          Солипсизм не признаёт существование Мира вне сознания (чьего либо). Я признаю существование Мира вне сознания и независимо от существования любых субъектов, но полагаю, что каждый из существующих субъектов воспринимает Мир по своему.

          Мир первичен, любой субъект вторичен

          наука физика к восприятию отношения не имеет, она основана на показаниях приборов

          а приборы существуют для того чтобы их кто то воспринимал

          Мне нравится комментарий https://habr.com/ru/users/Akabolka/comments/ пожалуйста посмотрите


  1. SVT-RD
    23.04.2024 19:00
    +3

    Насколько я вижу, поднятая вами тема уже давно, с 70ых годов обсуждается и в философии и в теоретической физике как проблема "точной настройки" - или fine-tuning.
    Из англ вики:
    "In theoretical physicsfine-tuning is the process in which parameters of a model must be adjusted very precisely in order to fit with certain observations. This had led to the discovery that the fundamental constants and quantities fall into such an extraordinarily precise range that if it did not, the origin and evolution of conscious agents in the universe would not be permitted ".
    Из русскоязычной вики:
    "Причина тонкой настройки неизвестна, существует лишь несколько гипотетических объяснений. Ряд учёных и философов (Пол ДевисХью Росс[en]Ричард Суинбёрн и др.) полагает, что в основе существующих закономерностей лежит гипотетический разумный замысел. Другие учёные (Мартин РисЛеонард СасскиндВиктор СтенджерИгорь НовиковАндрей Линде и др.) считают тонкую настройку случайным образованием в гипотетической мультивселенной. Суть этой гипотезы состоит в том, что существует огромное множество (не менее 10500) вселенных (или областей мультивселенной) со всевозможными значениями констант и начальных условий. И естественно ожидать, что среди этого множества наверняка найдётся и такая вселенная, где значения констант подходят для образования звёзд, галактик и жизни. Существуют и другие предположения — в частности, более общая теория («теория всего») или космологический естественный отбор Ли Смолина. "
    Я бы еще добавил к объясняющим гипотезам и современные наработки команды теоретического физика Нимы Аркани Хамеда, которая обосновывает существование более фундаментальной, чем наша физическая, реальности, которая описывается позитивной геометрией, через амплитуэдр и т.п.


    1. Imaginarium
      23.04.2024 19:00

      То, что Вы перечисляете и разъясняете -- явный оверхед для автора, у него даже терминов нет, он пытается провести рассуждения на уровне обыденного языка и непрерывно путается.


      1. AleksandrTallinn Автор
        23.04.2024 19:00

        и непрерывно путается.

        а можно конкретные примеры?


  1. Korova-ugnetatel
    23.04.2024 19:00

    "вы кладете в коробки то 59, то 61 спичку - вы там что, все сумасшедшие?"

    Под конец комментов подключились спецы, стало не так горячо ( респект сепулькам!).

    А полное ощущение такое, как будто новый философский трактат прочитал и критику к нему. Просто поток горячечных мыслей и постоянные кивки на "икспертов" типа Ницше, Канта, Поппера, чтобы добавить убедительности своей точке зрения.


    1. AleksandrTallinn Автор
      23.04.2024 19:00

      поток горячечных мыслей 

      а можно конкретные примеры?


    1. AleksandrTallinn Автор
      23.04.2024 19:00

      "икспертов" типа Ницше, Канта, Поппера

      Круто вы их. А кто тогда по вашему не "иксперт" ? Кого вы читали в последнее время к примеру?


  1. DenisYahnovec
    23.04.2024 19:00
    +2

    Бегло прочел Вашу статью, сразу скажу спасибо за проделанную работу.

    Все таки Ваш материал больше похож на филосовское размышление, и почти половина материала об объективности и субъективности, главный вывод что мы не понимаем и не можем понять и т.д. к слову мы изучаем вселенную не только за счет наших чувств и мыслей а с помощью измерительных приборов которые состоят из атомов и молекул, они уже не так субъективны, Вы можете отправить эти инструменты в космос и смотреть от туда или закопать под землю и проводить эксперименты там, и Вы будете получать данные от этих экспериментов. например БАК (Большой адронный коллайдер) все таки в изучении этого мира у нас есть определенная объективность.

    Что касается эволюции жизни посмотрите материалы Александра Панова о предбиологической жизни и о том что по данным ученых жизнь на земле , бактериальная, появилась примерно 4-4,5 миллиарда лет назад примерно тогда когда сама планета земля и возникла, т.е. случайность ли это? а может жизнь уже была во вселенной и эволюционировала в газопылевом облаке из которого и сформировалась земля, марс и венера? да, а что если и там шла эволюция жизни???

    Говорить же и множестве вселенных о случайности жизни это примерно тоже самое что я скажу что Жизнь это закономерность, что вселенная циклична, как это проверить? хорошо, посмотрите на результаты телескопа Джеймса веба который увидел в ранней вселенной примерно 500 млн. лет от рождения уже сформировавшиеся галктики со сверхмассивными черными дырами, так и еще возможно были найдены следы таких элементов как углерод и кальций, т.е. остатков от взрывов звезд, а по текущим представлениям так рано таких объектов мы наблюдать не должны, что это? либо мы не так понимаем эволюцию вселенной, а что если эта галактика залетело с соседней вселенной? я же могу пофантазировать тоже, тогда получается что соседняя галактика имеет такие же физически законы и константы.... да и почему эти константы должны быть другими, проблема тонкой настройки вселенной, что если они все взаимосвязаны, ядро и электрон могут быть в корне иметь один источник... ну да ладно не буду начинать

    Вот посмотрите интересный материал

    От большого взрыва до разума - универсальная эволюция | Александр Панов

    https://www.youtube.com/watch?v=5KZQEfsQljk


  1. Akabolka
    23.04.2024 19:00
    +1

    А я согласен с автором публикации.Объективной реальности не существует по определению.Есть субъективная реальность в виде мыслей, эмоций, ВТО, ОСов, обычных снов, фантазий, грез...творчества и проч. и есть условно объективная, принятая большинством субъектов в качестве действующей Картины Мира.

    Хотим мы того или нет, но как только мы проводим наблюдение/измерение,то моментально привносим в наблюдаемое некие изменения/дополнения/искажения формируя некий субъективный образ в соотвествии с ранее принятыми убеждениями, моделями и стереотипами.

    Физиологическая ограниченность диапазона восприятия органов чувств, несовершенство приборов, интерпретации измерения, да и наша формальная логика, тяготеющая к бинарной, ДА-НЕТной форме, часто и густо не учитывающая 50 оттенков серого...всё это никак не похоже на объективное восприятие действительности.

    Хотим мы, к примеру, измерить силу тока в цепи.Подключаем к примеру аналоговый (стрелочный) или цифровой амперметр и меряем.Получаем 1 ампер и считаем, что объективно измерили силу тока, ссылаясь на независимый от нас, субъектов, вполне себе объективный амперметр.

    Но так ли это? А на отклонение стрелки указывающей величину тока ушла какая-то часть энергии? Безусловно.Нужно ее учитывать, что бы таки действительно абсолютно точно, т.е. объективно, произвести измерение? Включаем в цепь катушки на которую завязана стрелка, еще один амперметрик, что бы замерить ту часть, которая ушла на отклонение "главной" стрелки.Меряем.0.1 ампер.Ну и так далее, до бесконечности, мы будем " "откусывать" часть объективности...самим фактом измерения! Так о какой абсолютнрй истине или объективности может идти речь?

    Число Пи? В нашей метрике пространства- времени оно равно 3,14 и сколько там триллионов знаков после запятой уже навычисляли? А если в пространстве с другой метрикой, где-нибудь вблизи Великого Аттрактора, где кривизна пространства будет,к примеру, отличная от нашей, число Пи будет 3,5? Или вообще 4? Какое из Пи более объективно?

    Или возьмем, к примеру радугу.Существует ли она объективно? Т.е. без наблюдателя? Кем бы он ни был? Да хоть вообще всех живых существ уберем с планеты Земля! Уберем и...радуга исчезнет! Вот совсем ее не будет! Равно как и сейчас ее нет.Что бы радуга состоялась как объект наблюдения нужно три вещи: солнце, дождь и...наблюдатель с глазами как у человека! Т.е. субъект.

    "Существует ли Луна, пока мы на нее не смотрит" - вопрошал Эйштейн сотоварищи .Квантмех, в лице неравенств Белла, подтверженных не так давно экспериментально, однозначно отвечает, что нет! У Вселенной и ее частей (частиц) вообще нет никаких определенных свойств до тех пор, как мы эту частицу не поймали и не измерили те самые свойства.

    О какой объективности можно вообще говорить? Складывать 1+1 и получить 2? Че, серьезно? Кто-нибудь видел, что бы в природе, без человека, кто-то что-то считал,складывал, умножал и брал тройные интегралы? Высшая математика, хоть и царица наук, но это всего лишь абстрактный язык-мозгодробилка, который помогает привнести в обшую Картину Мира отдельные мелкие штрихи и мазки, интересные только ценителям "живописи" и так же как любой другой язык она если не субъективна, то максимум условно-объективна.

    Сори за простынь.

    Статья занимательная.Прочитал с удовольствием.


    1. phenik
      23.04.2024 19:00
      +2

      Объективной реальности не существует по определению. Есть субъективная реальность в виде мыслей, эмоций, ВТО, ОСов, обычных снов, фантазий, грез...творчества и проч. и есть условно объективная, принятая большинством субъектов в качестве действующей Картины Мира.

      Солипсизм рулит?) Что является источником новых знаний, которые не сводятся к существующим? Каким образом они появляются в физике?

      И. Кант в своей теории познания, утверждал, что объективная реальность состоит из "вещей в себе", т.е. вещей самих по себе, безотносительно к каким либо субъектам познания и по этой причине до конца непознаваемых. Субъектам они даны только, как феномены - "вещи для нас", и нет принципиальных ограничений их познания любыми эмпирическими и рациональными методами. Историческая динамика развития физических представлений подтверждает этот тезис в форме общенаучного методологического принципа соответствия (1, 2). Можно представить эту реальность этаким идеальным пределом (трансцендентным по определению Канта) безграничного процесса познания. Нужно только помнить, что этот путь познания не монотонный, как в случае математического предела. Принцип соответствия выполняется только для формальных моделей теорий, концептуальных нет. Концептуальные представления новых теорий могут впрямую не сводить к концептам старых. Общеизвестный пример - принцип суперпозиции в КМ, который не имеет непосредственного аналога в классической. Т.е. идейный путь познания извилист и непредсказуем, и определяется неожиданными, как правило случайными (классический пример - открытие радиоактивности), экспериментальными и наблюдательными открытиями, которые требуют разработки новых теорий для их объяснения, если в прямую противоречат существующим. Таковы проявления объективной реальности, как источника новых знаний.


      1. Akabolka
        23.04.2024 19:00
        +1

        Солипсизм рулит?) 

        Таки да, в части касающейся построения общепринятой Картины Мира - да, рулит.

        На ОВФ о первичности материи или сознания я лично отвечаю следующим образом. Да, материя, которая " под капотом" первична. Сознание - вторично.Однако первичность материи вовсе не означает, что то, что мы наблюдаем/воспринимаем является " всем что есть" и существующим независимо от нас.Это всего лишь картинка, наподобие радуги, созданная нашим способом восприятия.И в этом смысле, то, что мы привыкли считать Картиной Мира, Физической ареной,Тоналем ( назовите как угодно) является вторичным по отношению к сознанию.А кантовская "вещь в себе" как тот силикатный кирпич, который расколов надвое, мы увидим уже два полукирпича, а не его " внутренности", являет нам только лишь свою внешнюю сторону, оставляя внутренне содержание скрытым, т.е каковы они сами по себе, мы ничего не знаем: мы знаем их только как  явления отображенные нами в представениях, которые они в нас производят, воздействуя на наши органы чувств.

        Таким образом, напрашивается вывод, что бъективная реальность существует только лишь чисто умозрительно... т.е. при отсутствии наблюдателя.А с наблюдателем - она становится субъективной.И даже 100500 субъектов, которые согласились принимать число Пи равным 3,14, не делают его чем-то объективным, существующим независимо от нас. Что, разве мы с вами чем-то принципиально отличаемся друг от друга ощупывая слона с повязкой на глазах? Да ничем! Так какая разница между одним однотипным наблюдателем от другого? Как по мне -никакой.

        Я бы это сравнил с коллапсом волновой функции.Пока нет наблюдения мы имеем лишь вероятность нахождения микрочастицы в той или иной области пространства подчиняющуюся Нормальному Закону Распределения.Но как только за одной из щелей в знаменитом опыте Юнга мы воткнем наблюдателя...хвать! ан демонов-то и нету! происходит процесс нарушения когерентности, вызываемый взаимодействием квантовомеханической системы с окружающей средой.